Как найти вектор квадрата

Скалярное произведение векторов

Угол
между векторами 
 может
принимать значения от 0 до 180 градусов
(от 0 до 
 радиан)
включительно. Аналитически данный факт
записывается в виде двойного
неравенства: 
 либо 
 (в
радианах).

В
литературе значок угла 
 часто
пропускают и пишут просто 
.

Определение: Скалярным
произведением двух векторов 
 и 
 называется
ЧИСЛО, равное произведению длин этих
векторов на косинус угла между ними:

Обозначение: скалярное
произведение обозначается через 
 или
просто 
.

Результат
операции является ЧИСЛОМ
:
Умножается вектор на вектор, а получается
число. Действительно, если длины
векторов 
 –
это числа, косинус угла – число, то их
произведение 
 тоже
будет числом.

Пример

Найти
скалярное произведение векторов 
 и 
,
если 

Решение: 

Ответ: 

Угол между векторами и значение скалярного произведения

Длины
ненулевых векторов всегда положительны: 
,
поэтому знак может зависеть только от
значения косинуса.

1)
Если угол между
векторами острый
  (от
0 до 90 градусов), то 
,
и скалярное
произведение будет положительным

.
Особый случай: если векторы сонаправлены,
то угол между ними считается нулевым 
,
и скалярное произведение также будет
положительным. Поскольку 
,
то формула упрощается: 
.

2)
Если угол между
векторами тупой
  (от
90 до 180 градусов), то 
,
и, соответственно, скалярное
произведение отрицательно

.
Особый случай: если векторы направлены
противоположно
, то
угол между ними считается развёрнутым
 (180
градусов). Скалярное произведение тоже
отрицательно, так как 

3)
Если угол между
векторами прямой
 (90
градусов), то 
 и скалярное
произведение равно нулю

.

Скалярное
произведение двух векторов равно нулю
тогда и только тогда, когда данные
векторы ортогональны
.
Короткая математическая запись: 

Третий
случай имеет большую практическую
значимость
, поскольку
позволяет проверить, ортогональны
векторы или нет.

Скалярный квадрат вектора Что будет, если вектор умножить на самого себя?

Или: 

Число 
 называется скалярным
квадратом
 вектора 
,
и обозначатся как 
.

Таким
образом, скалярный
квадрат вектора
 
 равен
квадрату длины данного вектора:

Из
данного равенства можно получить формулу
для вычисления длины вектора:

Свойства
скалярного произведения
.

Для
произвольных векторов 
 и
любого числа 
 справедливы
следующие свойства:

1) 
 –
переместительный или коммутативный закон
скалярного произведения.

2) 
 –
распределительный или дистрибутивный закон
скалярного произведения. Попросту,
можно раскрывать скобки.

3) 
 –
сочетательный или ассоциативный закон
скалярного произведения. Константу
можно вынести из скалярного произведения.

Пример

Найти
скалярное произведение векторов 
 и 
,
если известно, что 
.

Решение: 

(1)
Подставляем выражения векторов 
.

(2)
Раскрываем скобки по правилу умножения
многочленов. Раскрыть скобки нам
позволяет дистрибутивное свойство
скалярного произведения.

(3)
В первом и последнем слагаемом компактно
записываем скалярные квадраты векторов: 
.
Во втором слагаемом используем
перестановочность скалярного
произведения: 
.

(4)
Приводим подобные слагаемые: 
.

(5)
В первом слагаемом используем формулу
скалярного квадрата 
,
о которой не так давно упоминалось. В
последнем слагаемом, соответственно,
работает та же штука: 
.
Второе слагаемое раскладываем по
стандартной формуле 
.

(6)
Подставляем данные условия 
,
и ВНИМАТЕЛЬНО проводим окончательные
вычисления.

Ответ: 

Пример

Найти
длину вектора 
,
если 
.

Решение:

 

(1)
Поставляем выражение вектора 
.

(2)
Используем формулу длины: 
,
при этом в качестве вектора «вэ» у нас
выступает целое выражение 
.

(3)
Используем школьную формулу квадрата
суммы 
.

(4)
Дальнейшее аналогично действиям из
двух предыдущих задач.

Ответ: 

Соседние файлы в предмете [НЕСОРТИРОВАННОЕ]

  • #
  • #
  • #
  • #
  • #
  • #
  • #
  • #
  • #
  • #
  • #

Вектор. Скалярное произведение векторов. Угол между векторами.

Скалярным произведением (или внутренним произведением) 2 векторов есть операция с двумя

векторами, итогом чего является число (скаляр), которое не зависит от системы координат и которое

характеризует длины векторов-сомножителей и угол между векторами.

Также скалярным произведением двух векторов называется число, которое

равно произведению модулей 2 векторов на косинус угла между векторами.

Скалярное произведение векторов формула:

Этой операции соответствует умножение длины вектора x на проекцию вектора y на вектор x. Эта

операция зачастую рассматривается как коммутативная и линейная по каждому из сомножителей.

Скалярное произведение векторов ,, обозначается так: (порядок записи сомножителей не имеет

значения, т.е. ).

Еще используются такие обозначения: , , .

В основном имеется ввиду, что скалярное произведение определено положительно, т.е.

при каждом . Если этого не иметь ввиду, то произведение зовется индефинитным

(неопределенным).

Если хотя бы один из 2 векторов или равен нулевому вектору (равен нулю), то .

Свойства скалярного произведения векторов.

1. – симметричность.

2. обозначается и зовется скалярный квадрат.

3. Если , то

4. Если и и и , то . Обратное утверждение тоже соответствует

5.

6.

7.

Если же векторы и заданы своими координатами: , , то: скалярное

произведение векторов, формула:

Формула для определения длины вектора:

Длина (модуль) вектора, с известными координатами, равен квадратному корню из суммы квадратов

Длина вектора , заданного своими координатами, равна:

Как определить угол между 2 векторами:

Как найти угол между двумя векторами , , формула:

Ежели угол меж двумя векторами острый, то их скалярное произведение имеет положительный знак; если

же угол между двумя векторами тупой, то их скалярное произведение имеет отрицательный знак.

Скалярное произведение двух ненулевых векторов равно нулю, тогда и только тогда, когда эти векторы

ортогональны.

Альтернативное определение скалярного произведения векторов (вычисление скалярного

произведения двух векторов, заданных своими координатами).

Вычислить координаты вектора, если заданы координаты его начала и его конца очень просто. Давайте

рассмотрим этот вопрос:

Пусть есть вектор AB, точка А – это начало вектора, а В – конец, и координаты этих точек приведены ниже:

Исходя из этого, координаты вектора АВ:

Точно так же и в двухмерном пространстве – разница в отсутствии третьих координат.

Итак, предположим, даны два вектора, которые заданы набором координат своих точек:

а) В двухмерном пространстве (плоскость):

Значит, скалярное произведение этих векторов вычислим по формуле:

б) В трехмерном пространстве:

Как и в двухмерном случае, скалярное произведение двух векторов вычисляем по формуле:

Скалярное произведение векторов

О чем эта статья:

11 класс, ЕГЭ/ОГЭ

Статья находится на проверке у методистов Skysmart.
Если вы заметили ошибку, сообщите об этом в онлайн-чат
(в правом нижнем углу экрана).

Основные определения

Система координат — способ определить положение и перемещение точки или тела с помощью чисел или других символов.

Координаты — это совокупность чисел, которые определяют положение какого-либо объекта на прямой, плоскости, поверхности или в пространстве. Как найти координаты точки мы рассказали в этой статье.

Скаляр — это величина, которая полностью определяется в любой координатной системе одним числом или функцией.

Вектор — направленный отрезок прямой, для которого указано, какая точка является началом, а какая — концом.

Вектор с началом в точке A и концом в точке B принято обозначать как →AB. Векторы также можно обозначать малыми латинскими буквами со стрелкой или черточкой над ними, вот так: →a.

Скалярное произведение — это операция над двумя векторами, результатом которой является скаляр, то есть число, которое не зависит от выбора системы координат.

Результат операции является число. То есть при умножении вектор на вектор получается число. Если длины векторов |→a|, |→b| — это числа, косинус угла — число, то их произведение |→a|*|→b|*cos∠(→a, →b) тоже будет числом.

Чтобы разобраться в теме этой статьи, нам еще нужно узнать особенности угла между векторами.

Угол между векторами

Угол между векторами ∠(→a, →b) может принимать значения от 0° до 180° градусов включительно. Аналитически это можно записать в виде двойного неравенства: 0°=

2. Если угол между векторами равен 90°, то такие векторы перпендикулярны друг другу.

3. Если векторы направлены в разные стороны, тогда угол между ними 180°.

Также векторы могут образовывать тупой угол. Это выглядит так:

Скалярное произведение векторов

Определение скалярного произведения можно сформулировать двумя способами:

Скалярное произведение двух векторов a и b дает в результате скалярную величину, которая равна сумме попарного произведения координат векторов a и b.

Скалярным произведением двух векторов a и b будет скалярная величина, равная произведению модулей этих векторов, умноженная на косинус угла между ними:

→a * →b = →|a| * →|b| * cosα

  • Алгебраическая интерпретация.
  • Что важно запомнить про геометрическую интерпретацию скалярного произведения:

    • Если угол между векторами острый и векторы ненулевые, то скалярное произведение положительно, то есть cosα > 0.
    • Если угол между векторами тупой и векторы ненулевые, то скалярное произведение отрицательно, так как cosα

    Скалярное произведение в координатах

    Вычисление скалярного произведения можно произвести через координаты векторов в заданной плоскости или в пространстве.

    Скалярным произведением двух векторов на плоскости или в трехмерном пространстве в прямоугольной системе координат называется сумма произведений соответствующих координат векторов →a и →b.

    То есть для векторов →a = (ax, ay), →b = (bx, by) на плоскости в прямоугольной декартовой системе координат формула для вычисления скалярного произведения имеет вид: (→a, →b) = ax*bx + ay*by

    А для векторов →a = (ax, ay, az), →b = (bx, by, bz) в трехмерном пространстве скалярное произведение в координатах находится так: (→a, →b) = ax*bx + ay*by + az*bz

    Докажем это определение:

      Сначала докажем равенства

    для векторов →a = (ax, ay), →b = (bx, by) на плоскости, заданных в прямоугольной декартовой системе координат.

    Отложим от начала координат (точка О) векторы →OB = →b = (bx, by) и →OA = →a = (ax, ay)

    Тогда, →AB = →OB – →OA = →b – →a = (bx – ax, by – ay)

    Будем считать точки О, А и В вершинами треугольника ОАВ. По теореме косинусов можно записать:

    то последнее равенство можно переписать так:

    а по первому определению скалярного произведения имеем

  • Вспомнив формулу вычисления длины вектора по координатам, получаем
  • Абсолютно аналогично доказывается справедливость равенств (→a, →b) = |→a|*|→b|*cos(→a, →b) = ax*bx + ay*by + ax*bz для векторов →a = (ax, ay, az), →b = (bx, by, bz), заданных в прямоугольной системе координат трехмерного пространства.
  • Формула скалярного произведения векторов в координатах позволяет заключить, что скалярный квадрат вектора равен сумме квадратов всех его координат: на плоскости (→a, →a) = ax2 + ay2 в пространстве (→a, →a) = ax2 + ay2 + az2.
  • Записывайтесь на наши курсы по математике для учеников с 1 по 11 классы!

    Формулы скалярного произведения векторов заданных координатами

    Формула скалярного произведения векторов для плоских задач

    В плоской задаче скалярное произведение векторов a = и b = можно найти по формуле:

    a * b = ax * bx + ay * by

    Формула скалярного произведения векторов для пространственных задач

    В пространственной задаче скалярное произведение векторов a = и b = можно найти по формуле:

    a * b = ax * bx + ay * by + az * bz

    Формула скалярного произведения n-мерных векторов

    В n-мерном пространстве скалярное произведение векторов a = и b = можно найти по формуле:

    a * b = a1 * b1 + a2 * b2 + . + an * bn

    Свойства скалярного произведения

    Свойства скалярного произведения векторов:

      Скалярное произведение вектора самого на себя всегда больше или равно нулю. В результате получается нуль, если вектор равен нулевому вектору.

    →0 * →0 = 0

    Скалярное произведение вектора самого на себя равно квадрату его модуля:

    →a * →a = →∣∣a∣∣2

    Операция скалярного произведения коммуникативна, то есть соответствует переместительному закону:

    →a * →b = →b * →a

    Операция скалярного умножения дистрибутивна, то есть соответствует распределительному закону:

    (→a + →b) * →c = →a * →c + →b * →c

    Сочетательный закон для скалярного произведения:

    (k * →a) * →b = k * (→a * →b)

    Если скалярное произведение двух ненулевых векторов равно нулю, то эти векторы ортогональны, то есть перпендикулярны друг другу:

    a ≠ 0, b ≠ 0, a * b = 0 a ┴ b

    Эти свойства очень легко обосновать, если отталкиваться от определения скалярного произведения в координатной форме и от свойств операций сложения и умножения действительных чисел.

    Для примера докажем свойство коммутативности скалярного произведения (→a, →b) = (→b, →a)

    По определению (→a, →b) = ax*bx + ay*by и (→b, →a) = bx*ax + by*ay. В силу свойства коммутативности операции умножения действительных чисел, справедливо ax*bx = bx*ax b ay*by = by*ay, тогда ax*bx + ay*by = bx*ax + by*ay.

    Следовательно, (→a, →b) = (→b, →a), что и требовалось доказать.

    Аналогично доказываются остальные свойства скалярного произведения.

    Следует отметить, что свойство дистрибутивности скалярного произведения справедливо для любого числа слагаемых, то есть,

    Примеры вычислений скалярного произведения

    Пример 1.

    Вычислите скалярное произведение двух векторов →a и →b, если их длины равны 3 и 7 единиц соответственно, а угол между ними равен 60 градусам.

    У нас есть все данные, чтобы вычислить скалярное произведение по определению:

    (→a,→b) = →|a| * →|b| * cos(→a,→b) = 3 * 7 cos60° = 3 * 7 * 1/2 = 21/2 = 10,5.

    Ответ: (→a,→b) = 21/2 = 10,5.

    Пример 2.

    Найти скалярное произведение векторов →a и →b, если →|a| = 2, →|b| = 5, ∠(→a,→b) = π/6.

    Используем формулу →a * →b = →|a| * →|b| * cosα.

    В данном случае:

    →a * →b = →|a| * →|b| * cosα = 2 * 5 * cosπ/6 = 10 * √3/2 = 5√3

    Пример 3.

    Как найти скалярное произведение векторов →a = 7*→m + 3*→n и →b = 5*→m + 8*→n, если векторы →m и →n перпендикулярны и их длины равны 3 и 2 единицы соответственно.

    По свойству дистрибутивности скалярного произведения имеем

    Сочетательное свойство позволяет нам вынести коэффициенты за знак скалярного произведения:

    В силу свойства коммутативности последнее выражение примет вид

    Итак, после применения свойств скалярного произведения имеем

    Осталось применить формулу для вычисления скалярного произведения через длины векторов и косинус угла между ними:

    Пример 4.

    В правильной треугольной призме ABCA1B1C1, все ребра которой равны 1, найти косинус угла между прямыми AB1 и BC1.

      Введем систему координат.

    Если сделать выносной рисунок основания призмы, получим понятный плоскостной рисунок с помощью которого можно легко найти координаты всех интересующих точек.

  • Точка А имеет координаты (0;0;0). Точка С — (1;0;0). Точка В — (1/2;√3/2;0). Тогда точка В1 имеет координаты (1/2;√3/2;1), а точка С1 – (1;0;1).
  • Найдем координаты векторов →AB1 и →BC1:
  • Найдем длины векторов →AB1 и →BC1:
  • Найдем скалярное произведение векторов →AB1 и →BC1:
  • Найдем косинус угла между прямыми AB1 и BC1:
  • Пример 5.

    а) Проверить ортогональность векторов: →a(1; 2; -4) и →b(6; -1; 1) .

    б) Выяснить, будут ли перпендикулярными отрезки KL и MN, если K(3;5), L(-2;0), M(8;-1), N(1;4).

    а) Выясним, будут ли ортогональны пространственные векторы. Вычислим их скалярное произведение: →ab = 1*6 + 2*(-1) + (-4)*1 = 0, следовательно

    б) Здесь речь идёт об обычных отрезках плоскости, а задача всё равно решается через векторы. Найдем их: →KL(-2-3; 0-5) = →KL(-5; -5), →MN(1-8; 4-(-1)) = →MN(-7;5)

    Вычислим их скалярное произведение: →KL*→MN = -5*(-7) + (-5)*5 = 10 ≠ 0, значит, отрезки KL и MN не перпендикулярны.

    Обратите внимание на два существенных момента:

    • В данном случае нас не интересует конкретное значение скалярного произведения, важно, что оно не равно нулю.
    • В окончательном выводе подразумевается, что если векторы не ортогональны, значит, соответствующие отрезки тоже не будут перпендикулярными. Геометрически это очевидно, поэтому можно сразу записывать вывод об отрезках, что они не перпендикулярны.

    Ответ: а) →a перпендикулярно →b, б) отрезки KL, MN не перпендикулярны.

    Пример 6.

    Даны три вершины треугольника A(-1; 0), B(3; 2), C(5; -4). Найти угол при вершине B — ∠ABC.

    По условию чертеж выполнять не требуется, но для удобства можно сделать:

    Требуемый угол ∠ABC помечен зеленой дугой. Сразу вспоминаем школьное обозначение угла: ∠ABC — особое внимание на среднюю букву B — это и есть нужная нам вершина угла. Для краткости можно также записать просто ∠B.

    Из чертежа видно, что угол ∠ABC треугольника совпадает с углом между векторами →BA и →BC, иными словами: ∠ABC = ∠(→BA; →BC).

    Вычислим скалярное произведение:

    Вычислим длины векторов:

    Найдем косинус угла:

    Когда такие примеры не будут вызывать трудностей, можно начать записывать вычисления в одну строчку:

    Полученное значение не является окончательным, поэтому нет особого смысла избавляться от иррациональности в знаменателе.

    Найдём сам угол:

    Если посмотреть на чертеж, то результат действительно похож на правду. Для проверки угол также можно измерить и транспортиром.

    Ответ: ∠ABC = arccos(1/5√2) ≈1,43 рад. ≈ 82°

    Важно не перепутать, что в задаче спрашивалось про угол треугольника, а не про угол между векторами. Поэтому указываем точный ответ: arccos(1/5√2) и приближенное значение угла: ≈1,43 рад. ≈ 82°, которое легко найти с помощью калькулятора.

    А те, кому мало и хочется еще порешать, могут вычислить углы ∠A, ∠C, и убедиться в справедливости канонического равенства ∠A + ∠B + ∠C = 180°.

    Линейная алгебра для разработчиков игр

    Эта статья является переводом цикла из четырёх статей «Linear algebra for game developers», написанных David Rosen и посвящённых линейной алгебре и её применению в разработке игр. С оригинальными статьями можно ознакомиться тут: часть 1, часть 2, часть 3 и часть 4. Я не стал публиковать переводы отдельными топиками, а объединил все статьи в одну. Думаю, что так будет удобнее воспринимать материал и работать с ним. Итак приступим.

    Зачем нам линейная алгебра?

    Одним из направлений в линейной алгебре является изучение векторов. Если в вашей игре применяется позиционирование экранных кнопок, работа с камерой и её направлением, скоростями объектов, то вам придётся иметь дело с векторами. Чем лучше вы понимаете линейную алгебру, тем больший контроль вы получаете над поведением векторов и, следовательно, над вашей игрой.

    Что такое вектор?

    В играх вектора используются для хранения местоположений, направлений и скоростей. Ниже приведён пример двухмерного вектора:

    Вектор местоположения (также называемый «радиус-вектором») показывает, что человек стоит в двух метрах восточнее и в одном метре к северу от исходной точки. Вектор скорости показывает, что за единицу времени самолёт перемещается на три километра вверх и на два — влево. Вектор направления говорит нам о том, что пистолет направлен вправо.

    Как вы можете заметить, вектор сам по себе всего лишь набор цифр, который обретает тот или иной смысл в зависимости от контекста. К примеру, вектор (1, 0) может быть как направлением для оружия, как показано на картинке, так и координатами строения в одну милю к востоку от вашей текущей позиции. Или скоростью улитки, которая двигается вправо со скоростью в 1 милю в час (прим. переводчика: довольно быстро для улитки, 44 сантиметра в секунду).

    Важно отслеживать единицы измерения. Допустим у нас есть вектор V (3,5,2). Это мало что говорит нам. Три чего, пять чего? В нашей игре Overgrowth расстояния указываются в метрах, а скорости в метрах в секунду. Первое число в этом векторе — это направление на восток, второе — направление вверх, третье — направление на север. Отрицательные числа обозначают противоположные направления, на запад, вниз и на юг. Местоположение, определяемое вектором V (3,5,2), находится в трёх метрах к востоку, в пяти метрах вверху и в двух метрах к северу, как показано на картинке ниже.

    Итак, мы изучили основы работы с векторами. Теперь узнаем как вектора использовать.

    Сложение векторов

    Чтобы сложить вектора, нам надо просто сложить каждую их составляющую друг с другом. Например:

    (0, 1, 4) + (3, -2, 5) = (0+3, 1-2, 4+5) = (3, -1, 9)

    Зачем нам нужно складывать вектора? Наиболее часто сложение векторов в играх применяется для физического интегрирования. Любой физический объект будет иметь вектора для местоположения, скорости и ускорения. Для каждого кадра (обычно это одна шестидесятая часть секунды), мы должны интегрировать два вектора: добавить скорость к местоположению и ускорение к скорости.

    Давайте рассмотрим пример с прыжками Марио. Он начинает с позиции (0, 0). В момент начала прыжка его скорость (1, 3), он быстро двигается вверх и вправо. Его ускорение равно (0, -1), так как гравитация тянет его вниз. На картинке показано, как выглядит его прыжок, разбитый на семь кадров. Чёрным текстом показана его скорость в каждом фрейме.

    Давайте рассмотрим первые кадры поподробнее, чтобы понять как всё происходит.

    Для первого кадра, мы добавляем скорость Марио (1, 3) к его местоположению (0, 0) и получаем его новые координаты (1, 3). Затем мы складываем ускорение (0, -1) с его скоростью (1, 3) и получаем новое значение скорости Марио (1, 2).

    Делаем то-же самое для второго кадра. Добавляем скорость (1, 2) к местоположению (1, 3) и получаем координаты (2, 5). Затем добавляем ускорение (0, -1) к его скорости (1, 2) и получаем новую скорость (1, 1).

    Обычно игрок контролирует ускорение игрового персонажа с помощью клавиатуры или геймпада, а игра, в свою очередь, рассчитывает новые значения для скоростей и местоположения, используя физическое сложение (через сложение векторов). Это та-же задача, которая решается в интегральном исчислении, просто мы его сильно упрощаем для нашей игры. Я заметил, что мне намного проще внимательно слушать лекции по интегральному исчислению, думая о практическом его применении, которое мы только что описали.

    Вычитание векторов

    Вычитание рассчитывается по тому-же принципу что и сложение — вычитаем соответствующие компоненты векторов. Вычитание векторов удобно для получения вектора, который показывает из одного местоположения на другое. Например, пусть игрок находится по координатам (1, 2) с лазерным ружьём, а вражеский робот находится по координатам (4, 3). Чтобы определить вектор движения лазерного луча, который поразит робота, нам надо вычесть местоположение игрока из местоположения робота. Получаем:

    (4, 3) — (1, 2) = (4-1, 3-2) = (3, 1).

    Умножение вектора на скаляр

    Когда мы говорим о векторах, мы называем отдельные числа скалярами. Например (3, 4) — вектор, а 5 — это скаляр. В играх, часто бывает нужно умножить вектор на число (скаляр). Например, моделируя простое сопротивление воздуха путём умножения скорости игрока на 0.9 в каждом кадре. Чтобы сделать это, нам надо умножить каждый компонент вектора на скаляр. Если скорость игрока (10, 20), то новая скорость будет:

    0.9*(10, 20) = (0.9 * 10, 0.9 * 20) = (9, 18).

    Длина вектора

    Если у нас есть корабль с вектором скорости V (4, 3), нам также понадобится узнать как быстро он двигается, чтобы посчитать потребность в экранном пространстве или сколько потребуется топлива. Чтобы сделать это, нам понадобится найти длину (модуль) вектора V. Длина вектора обозначается вертикальными линиями, в нашем случае длина вектора V будет обозначаться как |V|.

    Мы можем представить V как прямоугольный треугольник со сторонами 4 и 3 и, применяя теорему Пифагора, получить гипотенузу из выражения: x 2 + y 2 = h 2

    В нашем случае — длину вектора H с компонентами (x, y) мы получаем из квадратного корня: sqrt(x 2 + y 2 ).

    Итак, скорость нашего корабля равна:

    |V| = sqrt(4 2 + 3 2 ) = sqrt(25) = 5

    Этот подход используется и для трёхмерных векторов. Длина вектора с компонентами (x, y, z) рассчитывается как sqrt(x 2 + y 2 + z 2 )

    Расстояние

    Если игрок P находится в точке (3, 3), а взрыв произошёл в точке E по координатам (1, 2), нам надо определить расстояние между игроком и взрывом, чтобы рассчитать степень ущерба, нанесённого игроку. Это легко сделать, комбинируя две вышеописанных операции: вычитание векторов и их длину.
    Мы вычитаем P — E, чтобы получить вектор между ними. А затем определяем длину этого вектора, что и даёт нам искомое расстояние. Порядок следования операндов тут не имеет значения, |E — P| даст тот-же самый результат.

    Расстояние = |P — E| = |(3, 3) — (1, 2)| = |(2, 1)| = sqrt(2 2 +1 2 ) = sqrt(5) = 2.23

    Нормализация

    Когда мы имеем дело с направлениями (в отличие от местоположений и скоростей), важно, чтобы вектор направления имел длину, равную единице. Это сильно упрощает нам жизнь. Например, допустим орудие развёрнуто в направлении (1, 0) и выстреливает снаряд со скоростью 20 метров в секунду. Каков в данном случае вектор скорости для выпущенного снаряда?

    Так как вектор направления имеет длину равную единице, мы умножаем направление на скорость снаряда и получаем вектор скорости (20, 0). Если-же вектор направления имеет отличную от единицы длину, мы не сможем сделать этого. Снаряд будет либо слишком быстрым, либо слишком медленным.

    Вектор с длиной равной единице называется «нормализованным». Как сделать вектор нормализованным? Довольно просто. Мы делим каждый компонент вектора на его длину. Если, к примеру, мы хотим нормализовать вектор V с компонентами (3, 4), мы просто делим каждый компонент на его длину, то есть на 5, и получаем (3/5, 4/5). Теперь, с помощью теоремы Пифагора, мы убедимся в том, что его длина равна единице:

    (3/5) 2 + (4/5) 2 = 9/25 + 16/25 = 25/25 = 1

    Скалярное произведение векторов

    Что такое скалярное произведение (записывается как •)? Чтобы рассчитать скалярное произведение двух векторов, мы должны умножить их компоненты, а затем сложить полученные результаты вместе

    (a1, a2) • (b1, b2) = a1b1 + a2b2

    Например: (3, 2) • (1, 4) = 3*1 + 2*4 = 11. На первый взгляд это кажется бесполезным, но посмотрим внимательнее на это:

    Здесь мы можем увидеть, что если вектора указывают в одном направлении, то их скалярное произведение больше нуля. Когда они перпендикулярны друг другу, то скалярное произведение равно нулю. И когда они указывают в противоположных направлениях, их скалярное произведение меньше нуля.
    В основном, с помощью скалярного произведения векторов можно рассчитать, сколько их указывает в одном направлении. И хоть это лишь малая часть возможностей скалярного произведения, но уже очень для нас полезная.

    Допустим у нас есть стражник, расположенный в G(1, 3) смотрящий в направлении D(1,1), с углом обзора 180 градусов. Главный герой игры подсматривает за ним с позиции H(3, 2). Как определить, находится-ли главный герой в поле зрения стражника или нет? Сделаем это путём скалярного произведения векторов D и V (вектора, направленного от стражника к главному герою). Мы получим следующее:

    V = H — G = (3, 2) — (1, 3) = (3-1, 2-3) = (2, -1)
    D•V = (1, 1) • (2, -1) = 1*2 + 1*-1 = 2-1 = 1

    Так как единица больше нуля, то главный герой находится в поле зрения стражника.

    Мы уже знаем, что скалярное произведение имеет отношение к определению направления векторов. А каково его более точное определение? Математическое выражение скалярного произведения векторов выглядит так:

    Где Θ (произносится как «theta») — угол между векторами A и B.

    Это позволяет нам найти Θ (угол) с помощью выражения:

    Как я говорил ранее, нормализация векторов упрощает нашу жизнь. И если A и B нормализованы, то выражение упрощается следующим образом:

    Давайте опять рассмотрим сценарий со стражником. Пусть теперь угол обзора стражника будет равен 120 градусам. Получим нормализованные вектора для направления взгляда стражника (D’) и для направления от стражника к главному герою (V’). Затем определим угол между ними. Если угол более 60 градусов (половина от угла обзора), то главный герой находится вне поля зрения стражника.

    D’ = D / |D| = (1, 1) / sqrt(1 2 + 1 2 ) = (1, 1) / sqrt(2) = (0.71, 0.71)
    V’ = V / |V| = (2, -1) / sqrt(2 2 + (-1) 2 ) = (2,-1) / sqrt(5) = (0.89, -0.45)

    Θ = acos(D’V’) = acos(0.71*0.89 + 0.71*(-0.45)) = acos(0.31) = 72

    Угол между центром поля зрения стражника и местоположением главного героя составляет 72 градуса, следовательно стражник его не видит.

    Понимаю, что это выглядит довольно сложно, но это потому, что мы всё делаем вручную. В программе это всё довольно просто. Ниже показано как я сделал это в нашей игре Overgrowth с помощью написанных мной С++ библиотек для работы с векторами:

    Векторное произведение

    Допустим у нас есть корабль с пушками, которые стреляют в правую и в левую стороны по курсу. Допустим, что лодка расположена вдоль вектора направления (2, 1). В каких направлениях теперь стреляют пушки?

    Это довольно просто в двухмерной графике. Чтобы повернуть направление на 90 градусов по часовой стрелке, достаточно поменять местами компоненты вектора, а затем поменять знак второму компоненту.
    (a, b) превращается в (b, -a). Следовательно у корабля, расположенного вдоль вектора (2, 1), пушки справа по борту будут стрелять в направлении (1, -2), а пушки с левого борта, будут стрелять в противоположном направлении. Меняем знаки у компонент вектора и получаем (-1, 2).

    А что если мы хотим рассчитать это всё для трехмерной графики? Рассмотрим пример с кораблём.
    У нас есть вектор мачты M, направленной прямо вверх (0, 1, 0) и направление ветра: север-северо-восток W (1, 0, 2). И мы хотим вычислить вектор направления паруса S, чтобы наилучшим образом «поймать ветер».

    Для решения этой задачи мы используем векторное произведение: S = M x W.

    Подставим теперь нужные нам значения:

    S = MxW = (0, 1, 0) x (1, 0, 2) = ([1*2 — 0*0], [0*1 — 0*2], [0*0 — 1*1]) = (2, 0, -1)

    Для расчётов вручную довольно сложно, но для графических и игровых приложений я рекомендую написать функцию, подобную той, что указана ниже и не вдаваться более в детали подобных расчётов.

    Векторное произведение часто используется в играх, чтобы рассчитать нормали к поверхностям. Направления, в которых «смотрит» та или иная поверхность. Например, рассмотрим треугольник с векторами вершин A, B и С. Как мы найдем направление в котором «смотрит» треугольник, то есть направление перпендикулярное его плоскости? Это кажется сложным, но у нас есть инструмент для решения этой задачи.

    Используем вычитание, для определения направления из A в С (C — A), пусть это будет «грань 1» (Edge 1) и направление из A в B (B — A), пусть это будет «грань 2» (Edge 2). А затем применим векторное произведение, чтобы найти вектор, перпендикулярный им обоим, то есть перпендикулярный плоскости треугольника, также называемый «нормалью к плоскости».

    Вот так это выглядит в коде:

    В играх основное выражение освещённости записывается как N • L, где N — это нормаль к освещаемой поверхности, а L — это нормализованный вектор направления света. В результате поверхность выглядит яркой, когда на неё прямо падает свет, и тёмной, когда этого не происходит.

    Теперь перейдем к рассмотрению такого важного для разработчиков игр понятия, как «матрица преобразований» (transformation matrix).

    Для начала изучим «строительные блоки» матрицы преобразований.

    Базисный вектор

    Допустим мы пишем игру Asteroids на очень старом «железе» и нам нужен простой двухмерный космический корабль, который может свободно вращаться в своей плоскости. Модель корабля выглядит так:

    Как нам рисовать корабль, когда игрок поворачивает его на произвольный градус, скажем 49 градусов против часовой стрелки. Используя тригонометрию, мы можем написать функцию двухмерного поворота, которая принимает координаты точки и угол поворота, и возвращает координаты смещённой точки:

    Применяя эту функцию ко всем трём точкам, мы получим следующую картину:

    Операции с синусами и косинусами работают довольно медленно, но так как мы делаем расчёты лишь для трёх точек, это будет нормально работать даже на старом «железе» (прим. переводчика: в случаях, когда предполагается интенсивное использование тригонометрических функций, для ускорения вычислений, в памяти организуют таблицы значений для каждой функции и рассчитывают их во время запуска приложения. Затем при вычислении той или иной тригонометрической функции просто производится обращение к таблице).

    Пусть теперь наш корабль выглядит вот так:

    Теперь старый подход будет слишком медленным, так как надо будет поворачивать довольно большое количество точек. Одно из элегантных решений данной проблемы будет звучать так — «Что если вместо поворота каждой точки модели корабля, мы повернём координатную решётку нашей модели?»

    Как это работает? Давайте посмотрим внимательнее, что собой представляют координаты.
    Когда мы говорим о точке с координатами (3, 2), мы говорим, что её местоположение находится в трех шагах от точки отсчёта по координатной оси X, и двух шагах от точки отсчёта по координатной оси Y.

    По-умолчанию координатные оси расположены так: вектор координатной оси X (1, 0), вектор координатной оси Y (0, 1). И мы получим расположение: 3(1, 0) + 2(0, 1). Но координатные оси не обязательно должны быть в таком положении. Если мы повернём координатные оси, в это-же время мы повернём все точки в координатной решётке.

    Чтобы получить повернутые оси X и Y мы применим тригонометрические функции, о которых говорили выше. Если мы поворачиваем на 49 градусов, то новая координатная ось X будет получена путём поворота вектора (0, 1) на 49 градусов, а новая координатная ось Y будет получена путём поворота вектора (0, 1) на 49 градусов. Итак вектор новой оси X у нас будет равен (0.66, 0.75), а вектор новой оси Y будет (-0.75, 0.66). Сделаем это вручную для нашей простой модели из трёх точек, чтобы убедиться, что это работает так, как нужно:

    Координаты верхней точки (0, 2), что означает, что её новое местоположение находится в 0 на новой (повёрнутой) оси X и 2 на новой оси Y:

    0*(0.66,0.75) + 2*(-0.75, 0.66) = (-1.5, 1.3)

    Нижняя левая точка (-1, -1), что означает, что её новое местоположение находится в -1 на повернутой оси X, и -1 на повернутой оси Y:

    -1*(0.66,0.75) + -1*(-0.75, 0.66) = (0.1, -1.4)

    Нижняя правая точка (1, -1), что означает её новое местоположение находится в 1 на повернутой оси X, и -1 на повернутой оси Y

    1*(0.66,0.75) + -1*(-0.75, 0.66) = (1.4, 0.1)

    Мы показали, как координаты корабля отображаются в другой координатной сетке с повернутыми осями (или «базисными векторами»). Это удобно в нашем случае, так как избавляет нас от необходимости применять тригонометрические преобразования к каждой из точек модели корабля.

    Каждый раз, когда мы изменяем базисные вектора (1, 0) и (0, 1) на (a, b) и (c, d), то новая координата точки (x, y) может быть найдена с помощью выражения:

    Обычно базисные вектора равны (1, 0) и (0, 1) и мы просто получаем x(1, 0) + y(0, 1) = (x, y), и нет необходимости заботиться об этом дальше. Однако, важно помнить, что мы можем использовать и другие базисные вектора, когда нам это нужно.

    Матрицы

    Матрицы похожи на двухмерные вектора. Например, типичная 2×2 матрица, может выглядеть так:

    Когда вы умножаете матрицу на вектор, вы суммируете скалярное произведение каждой строки с вектором, на который происходит умножение. Например, если мы умножаем вышеприведённую матрицу на вектор (x, y), то мы получаем:

    Будучи записанным по-другому, это выражение выглядит так:

    Выглядит знакомо, не так-ли? Это в точности такое-же выражение, которые мы использовали для смены базисных векторов. Это означает, что умножая 2×2 матрицу на двухмерный вектор, мы тем самым меняем базисные вектора. Например, если мы вставим стандартные базисные вектора в (1, 0) и (0, 1) в колонки матрицы, то мы получим:

    Это единичная матрица, которая не даёт эффекта, который мы можем ожидать от нейтральных базисных векторов, которые мы указали. Если-же мы повернём базисные вектора на 49-градусов, то мы получим:

    Эта матрица будет поворачивать двухмерный вектор на 49 градусов против часовой стрелки. Мы можем сделать код нашей игры Asteriods более элегантным, используя матрицы вроде этой. Например, функция поворота нашего корабля может выглядеть так:

    Однако, наш код будет ещё более элегантным, если мы сможем также включить в эту матрицу перемещение корабля в пространстве. Тогда у нас будет единая структура данных, которая будет заключать в себе и применять информацию об ориентации объекта и его местоположении в пространстве.

    К счастью есть способ добиться этого, хоть это и выглядит не очень элегантно. Если мы хотим переместиться с помощью вектора (e, f), мы лишь включаем его в нашу матрицу преобразования:

    И добавляем дополнительную единицу в конец каждого вектора, определяющего местоположение объекта, например так:

    Теперь, когда мы перемножаем их, мы получаем:

    (a, c, e) • (x, y, 1) + (b, d, f) • (x, y, 1) + (0, 0, 1) • (x, y, 1)

    Что, в свою очередь, может быть записано как:

    x(a, b) + y(c, d) + (e, f)

    Теперь у нас есть полный механизм трансформации, заключённый в одной матрице. Это важно, если не принимать в расчёт элегантность кода, так как с ней мы теперь можем использовать все стандартные манипуляции с матрицами. Например перемножить матрицы, чтобы добавить нужный эффект, или мы можем инвертировать матрицу, чтобы получить прямо противоположное положение объекта.

    Трехмерные матрицы

    Матрицы в трехмерном пространстве работают так-же как и в двухмерном. Я приводил примеры с двухмерными векторами и матрицами, так как их просто отобразить с помощью дисплея, показывающего двухмерную картинку. Нам просто надо определить три колонки для базисных векторов, вместо двух. Если базисные вектора это (a,b,c), (d,e,f) and (g,h,i) то наша матрица будет выглядеть так:

    Если нам нужно перемещение (j,k,l), то мы добавляем дополнительную колонку и строку, как говорили раньше:

    И добавляем единицу [1] в вектор, как здесь:

    Вращение в двухмерном пространстве

    Так как в нашем случае у нас только одна ось вращения (расположенная на дисплее), единственное, что нам надо знать, это угол. Я говорил об этом ранее, упоминая, что мы можем применять тригонометрические функции для реализации функции двухмерного вращения наподобие этой:

    Более элегантно это можно выразить в матричной форме. Чтобы определить матрицу, мы можем применить эту функцию к осям (1, 0) и (0, 1) для угла Θ, а затем включить полученные оси в колонки нашей матрицы. Итак, начнём с координатной оси X (1, 0). Если мы применим к ней нашу функцию, мы получим:

    (1*cos(Θ) — 0*sin(Θ), 1*sin(Θ) + 0*cos(Θ)) = (cos(Θ), sin(Θ))

    Затем, мы включаем координатную ось Y (0, 1). Получим:

    (0*cos(Θ) — 1*sin(Θ), 0*sin(Θ) + 1*cos(Θ)) = (-sin(Θ), cos(Θ))

    Включаем полученные координатные оси в матрицу, и получаем двухмерную матрицу вращения:

    Применим эту матрицу к Сюзанне, мартышке из графического пакета Blender. Угол поворота Θ равен 45 градусов по часовой стрелке.

    Как видите — это работает. Но что если нам надо осуществить вращение вокруг точки, отличной от (0, 0)?
    Например, мы хотим вращать голову мартышки вокруг точки, расположенной в её ухе:

    Чтобы сделать это, мы можем начать с создания матрицы перемещения (translation matrix) T, которая перемещает объект из начальной точки в точку вращения в ухе мартышки, и матрицу вращения R, для вращения объекта вокруг начальной точки. Теперь для вращения вокруг точки, расположенной в ухе, мы можем сперва переместить точку в ухе на место начальной точки, с помощью инвертирования матрицы T, записанной как T -1 . Затем, мы вращаем объект вокруг начальной точки, с помощью матрицы R, а затем применяем матрицу T для перемещения точки вращения назад, к своему исходному положению.
    Ниже дана иллюстрация к каждому из описанных шагов:

    Это важный шаблон, который мы будем применять позднее — применение вращения для двух противоположных трансформаций позволяет нам вращать объект в другом «пространстве». Что очень удобно и полезно.

    Теперь рассмотрим трёхмерное вращение.

    Трёхмерное вращение

    Вращение вокруг оси Z работает по тому-же принципу, что и вращение в двухмерном пространстве. Нам лишь нужно изменить нашу старую матрицу, добавив к ней дополнительную колонку и строку:

    Применим эту матрицу к трехмерной версии Сюзанны, мартышки из пакета Blender. Угол поворота Θ пусть будет равен 45 градусов по часовой стрелке.

    То-же самое. Вращение только вокруг оси Z ограничивает нас, как насчёт вращения вокруг произвольной оси?

    Вращение, определяемое осью и углом (Axis-angle rotation)

    Представление вращения, определяемого осью и углом, также известно как вращение в экспоненциальных координатах, параметризованное вращением двух величин. Вектора, определяющего вращение направляющей оси (прямая линия) и угла, описывающего величину поворота вокруг этой оси. Вращение осуществляется согласно правилу правой руки.

    Итак, вращение задаётся двумя параметрами (axis, angle), где axis — вектор оси вращения, а angle — угол вращения. Этот приём довольно прост и являет собой отправную точку для множества других операций вращения, с которыми я работаю. Как практически применить вращение, определяемое осью и углом?

    Допустим мы имеем дело с осью вращения, показанной на рисунке ниже:

    Мы знаем как вращать объект вокруг оси Z, и мы знаем как вращать объект в других пространствах. Итак, нам лишь надо создать пространство, где наша ось вращения будет являться осью Z. И если эта ось будет осью Z, то что будет являться осями X и Y? Займемся вычислениями сейчас.

    Чтобы создать новые оси X и Y нам нужно лишь выбрать два вектора, которые перпендикулярны новой оси Z и перпендикулярны друг другу. Мы уже говорили ранее о векторном умножении, которое берёт два вектора и даёт в итоге перпендикулярный им вектор.

    У нас есть один вектор сейчас, это ось вращения, назовём его A. Возьмём теперь случайный другой вектор B, который находится не в том-же направлении, что и вектор A. Пусть это будет (0, 0, 1) к примеру.

    Теперь мы имеем ось вращения A и случайный вектор B, мы можем получить нормаль C, через векторное произведение A и B. С перпендикулярен векторам A и B. Теперь мы делаем вектор B перпендикулярным векторам A и C через их векторное произведение. И всё, у нас есть все нужные нам оси координат.

    На словах это звучит сложно, но довольно просто выглядит в коде или будучи показанным в картинках.
    Ниже показано, как это выглядит в коде:

    Тут показана иллюстрация для каждого шага:

    Теперь, имея информацию о новых координатных осях, мы можем составить матрицу M, включив каждую ось как колонку в эту матрицу. Нам надо убедиться, что вектор A является третьей колонкой, чтобы он был нашей новой осью координат Z.

    Теперь это похоже на то, что мы делали для поворота в двухмерном пространстве. Мы можем применить инвертированную матрицу M, чтобы переместиться в новую систему координат, затем произвести вращение, согласно матрице R, чтобы повернуть объект вокруг оси Z, затем применить матрицу M, чтобы вернуться в исходное координатное пространство.

    Теперь мы можем вращать объект вокруг произвольной оси. В конце концов мы можем просто создать матрицу T = T = M -1 RM и использовать её много раз, без дополнительных усилий с нашей стороны. Есть более эффективные способы конвертирования вращений, определяемых осью и углом во вращения, определяемые матрицами. Просто описанный нами подход показывает многое из того, о чём мы говорили ранее.

    Вращение, определяемое осью и углом, возможно, самый интуитивно понятный способ. Применяя его, очень легко инвертировать поворот, поменяв знак у угла, и легко интерполировать, путём интерполяции угла. Однако тут есть серьёзное ограничение, и заключается оно в том, что такое вращение не является суммирующим. То есть вы не можете комбинировать два вращения, определяемых осью и углом в третье.
    Вращение, определяемое осью и углом — хороший способ для начала, но оно должно быть преобразовано во что-то другое, чтобы использоваться в более сложных случаях.

    Эйлеровские углы

    Эйлеровские углы представляют собой другой способ вращения, заключающийся в трёх вложенных вращениях относительно осей X, Y и Z. Вы, возможно, сталкивались с их применением в играх, где камера показывает действие от первого лица, либо от третьего лица.

    Допустим вы играете в шутер от первого лица и вы повернулись на 30 градусов влево, а затем посмотрели на 40 градусов вверх. В конце-концов в вас стреляют, попадают, и, в результате удара, камера поворачивается вокруг своей оси на 45 градусов. Ниже показано вращение с помощью углов Эйлера (30, 40, 45).

    Углы Эйлера — удобное и простое в управлении средство. Но у этого способа есть два недостатка.

    Первый, это вероятность возникновения ситуации под названием «блокировка оси» или «шарнирный замок» (gimbal lock). Представьте, что вы играете в шутер от первого лица, где вы можете посмотреть влево, вправо, вверх и вниз или повернуть камеру вокруг зрительной оси. Теперь представьте, что вы смотрите прямо вверх. В этой ситуации попытка взглянуть налево или направо будет аналогична попытке вращения камеры. Всё что мы можем вы этом случае, это вращать камеру вокруг своей оси, либо посмотреть вниз. Как вы можете представить, это ограничение делает непрактичным применение углов Эйлера в лётных симуляторах.

    Второе — интерполяция между двумя эйлеровскими углами вращения не даёт кратчайшего пути между ними.
    Например, у вас две интерполяции между двумя одинаковыми вращениями. Первая использует интерполяцию эйлеровского угла, вторая использует сферическую линейную интерполяцию (spherical linear interpolation (SLERP)), чтобы найти кратчайший путь.

    Итак, что-же больше подойдет для интерполяции вращений? Может быть матрицы?

    Вращение с помощью матриц

    Как мы уже говорили ранее, матрицы вращения хранят в себе информацию о трёх осях. Это означает, что интерполяция между двумя матрицами лишь линейно интерполирует каждую ось. В результате это даёт нам эффективный путь, то так-же привносит новые проблемы. Например, тут показаны два вращения и одно интерполированное полу-вращение:

    Как вы можете заметить, интерполированное вращение значительно меньше, чем любое из исходных вращений, и две оси более не перпендикулярны друг другу. Это логично, если вдуматься — середина отрезка, соединяющего любые две точки на сфере будет расположена ближе к центру сферы.

    Это в свою очередь порождает известный «эффект фантика» (candy wrapper effect), при применении скелетной анимации. Ниже показана демонстрация этого эффекта на примере кролика из нашей игры Overgrowth (прим. переводчика: обратите внимание на середину туловища кролика).

    Вращение, основанное на матричных операциях, очень полезно, так как они могут аккумулировать вращения без всяких проблем, вроде блокировки оси (gimbal lock), и может очень эффективно применяться к точкам сцены. Вот почему поддержка вращения на матрицах встроена в графические карты. Для любого типа трёхмерной графики матричный формат вращения — это всегда итоговый применяемый способ.

    Однако, как мы уже знаем, матрицы не очень хорошо интерполируются, и они не столь интуитивно понятны.

    Итак, остался только один главный формат вращения. Последний, но тем не менее, важный.

    Кватернионы

    Что-же такое кватернионы? Если очень кратко, то это альтернативный вариант вращения, основанный на оси и угле (axis-angle rotation), который существует в пространстве.

    Подобно матрицам они могут аккумулировать вращения, то есть вы можете составлять из них цепочку вращений, без опаски получить блокировку оси (gimbal lock). И в то-же время, в отличие от матриц, они могут хорошо интерполироваться из одного положения в другое.

    Являются-ли кватернионы лучшим решением, нежели остальные способы вращений (rotation formats)?
    На сегодняшний день они комбинируют все сильные стороны других способов вращений. Но у них есть два слабых места, рассмотрев которые, мы придём к выводу, что кватернионы лучше использовать для промежуточных вращений. Итак, каковы недостатки кватернионов.

    Во-первых кватернионы непросто отобразить на трёхмерном пространстве. И мы вынуждены всегда реализовывать вращение более простым способом, а затем конвертировать его. Во-вторых, кватернионы не могут эффективно вращать точки, и мы вынуждены конвертировать их в матрицы, чтобы повернуть значительное количество точек.

    Это означает, что вы скорее всего не начнете или не закончите серию вращений с помощью кватернионов. Но с их помощью можно реализовать промежуточные вращения более эффективно, нежели при применении любого другого подхода.

    «Внутренняя кухня» механизма кватернионов не очень понятна и не интересна мне. И, возможно, не будет интересна и вам, если только вы не математик. И я советую вам найти библиотеки, которые работают с кватернионами, чтобы облегчить вам решение ваших задач с их помощью.

    Математические библиотеки «Bullet» или «Blender» будут хорошим вариантом для начала.

    [spoiler title=”источники:”]

    http://skysmart.ru/articles/mathematic/skalyarnoe-proizvedenie-vektorov

    http://habr.com/ru/post/131931/

    [/spoiler]

    Содержание:

    1. Векторы
    2. Действия над векторами
    3. Умножение вектора на число
    4. Скалярное произведение векторов
    5. Векторное произведение
    6. Смешенное произведение векторов
    7. Разложение вектора по базису
    8. Действия над векторами, заданными своими координатами
    9. Проекция вектора на ось
    10. Проекции вектора на оси координат
    11. Направляющие косинусы вектора
    12. Разложение вектора по ортам
    13. Действия над векторами, заданными в координатной форме
    14. Вектор – основные определения
    15. Операции над векторами и их свойства
    16. Сформулируем и докажем ещё одну важную для решения некоторых задач теорему.
    17. Координаты вектора
    18. Скалярное произведение векторов и его свойства
    19. Векторы и их решение
    20. Собственные числа и собственные векторы
    21. Векторная алгебра
    22. Векторы: основные определения, линейные операции
    23. Линейные операции над векторами
    24. Умножения вектора на скаляр
    25. Основные свойства проекции вектора на ось
    26. Прямоугольная система координат в пространстве. Координатная и алгебраическая формы задания векторов
    27. Скалярное, векторное, смешанное произведения векторов
    28. Векторное произведение двух векторов
    29. Смешанное произведение векторов, заданных в координатной форме
    30. Простейшие задачи аналитической геометрии
    31. Задача об определении площади треугольника
    32. Задача о деление отрезка в заданном отношении

    Векторы

    В математике вектором называют величину, которая характеризуется только числом и направлением. Так определённые векторы ещё называют свободными векторами. Примером физических величин, которые имеют векторный характер являются скорость, сила, ускорение. Геометрически вектор — это направленный отрезок, хотя правильней говорить про целый класс направленных отрезков, которые все параллельны между собой, имеют одинаковые длину и направление.

    Векторы

    Векторы обозначают малыми латинскими буквами с чертой сверху Векторы, или двумя большими латинскими буквами, которые обозначают его начало и конец, например  Векторы. Длина (модуль) вектора — это длина отрезка, который отвечает данному вектору и обозначается Векторы В зависимости от соотношения длин и направлений различают следующие виды векторов:

    Векторы

    Векторы

    Действия над векторами

    Рассмотрим основные действия, определённые над векторами.

    1. Сложение векторов. Суммой векторов Векторы называют вектор Векторы, который соединяет начало вектора Векторы с концом вектора Векторы, при условии, что вектор Векторы отложен от конца вектора Векторы. Такой способ сложения векторов называют правилом треугольника.

    Векторы

    Учитывая, что Векторы, то найти сумму векторов Векторы можно также по так называемым “правилом параллелограмма” (рис. 3)

    Векторы

    Вычитание векторов сводится к сложению противоположного вектора

    Векторы

    Запишем основные свойства действий сложения векторов:

     Векторы

    Заметим, что сумма нескольких векторов находится последовательным сложением двух из них, например:

    Векторы

    Геометрически сумма нескольких векторов находится их последовательным отложением один за одним так, чтоб начало следующего совпадало с концом предыдущего. Суммой является вектор, который будет соединять начало первого с концом последнего (рис. 4). Если такая последовательность векторов даёт замкнутую ломаную то суммой векторов является Векторы (рис. 5).

    Векторы

    Умножение вектора на число

    Произведением вектора Векторы на число Векторы называют вектор Векторы, для которого выполняются условия:

    а) Векторы;

    б) Векторы, причём Векторы сонаправленные если Векторы противоположно направленные, если Векторы. Отсюда, очевидно, что необходимым и достаточным условием коллинеарности векторов является соотношение Векторы.

    Запишем основные свойства действий умножения вектора на число:

    Векторы

    Скалярное произведение векторов

    Скалярным произведением Векторы или Векторы векторов Векторы и Векторы называют выражение Векторы, где Векторы угол, который образуют векторы. Отметим, что углом между векторами считают угол между их направлениями. Если хотя бы один из векторов равен Векторы, то их скалярное произведение считают равным нулю.

    Очевидно, что скалярное произведение двух ненулевых векторов будет равно нулю тогда и только тогда когда эти вектора перпендикулярны (ортогональны). Действительно, если Векторы. Но Векторы, следовательно,

    Векторы

    Наоборот, если Векторы и согласно определениям

    Векторы.

    Например, скалярное произведение Векторы будет равным

    Векторы

    Запишем основные свойства действий скалярного умножения векторов:

    Векторы

    Векторное произведение

    Векторным произведением Векторы двух векторов Векторы и Векторы называется вектор Векторы, который удовлетворяет условия:

    1) модуль вектора Векторы равен произведению модулей векторов  Векторы и Векторы на синус угла между ними

     Векторы

    2) вектор Векторы перпендикулярный к плоскости, которая определяется векторами Векторы и Векторы (рис. 5).

    3) вектор Векторы направленный так, что кратчайший поворот вектора Векторы к вектору Векторы видно с конца вектора Векторы таким, что происходит против движения стрелки (то есть вектора ВекторыВекторы и  образуют правую упорядоченную тройку, или правый руль).

    Векторы

    Модуль векторного произведения равен площади параллелограмма, построенного на векторах Векторы и Векторы. Векторное произведение выражается формулой Векторы, где Векторы площадь параллелограмма построенного на векторах Векторы и ВекторыВекторы единичный вектор направления Векторы.

    Приведём основные свойства векторного произведения:

    1) векторное произведение Векторы равно нулю, если векторы  Векторы и Векторы коллинеарные, или один из них нулевой;

    2) от перестановки местами векторов-сомножителей векторное произведение меняет знак на противоположный: Векторы (векторное произведение не имеет свойств перестановки);

    3) Векторы (распределительный закон);

    4) Векторы (соединительный закон).

    Физическое содержание векторного произведения такое. Если Векторы сила, а Векторы радиус-вектор точки её приложения, которая имеет начало в точке Векторы, то моментом силы Векторы относительно точки Векторы является вектор, который равен векторному произведению Векторы на Векторы, то есть Векторы.

    Смешенное произведение векторов

    Смешенным произведением векторов Векторы называют скалярное произведение вектора Векторы на вектор Векторы. Смешенное произведение обозначают (Векторы), поэтому по определению имеем

    Векторы

    Как результат скалярного произведения векторов Векторы и Векторы смешенное произведение является скалярной величиной (числом). Геометрически смешенное произведение — это объём параллелепипеда, построенного на эти векторах, взятый со знаком плюс, если векторы Векторы образуют правую тройку, и со знаком минус, когда эта тройка левая      (рис. 7).

     Векторы

    Действительно, Векторы, где Векторы угол между векторами Векторы угол между векторами Векторы и Векторы.

    Объём V параллелепипеда, построенного на векторах Векторы равный произведению площади основы S на высоту h.

    Векторы

    Однако, знак смешенного произведения совпадает со знаком Векторы, то есть он положительный, когда угол Векторы острый (Векторы образуют правую тройку векторов) и отрицательный, когда угол Векторы тупой (Векторы образуют левую тройку векторов). Поэтому:

    Векторы

    Из геометрического содержания смешенного произведения выходит, что 

    1) смешанное произведение равно нулю тогда и только тогда, когда перемноженные вектора копланарные (условие компланарных векторов);

    2) Векторы

    Учитывая коммутативность скалярного произведения и антикоммутативность векторного, для произвольных векторов Векторы имеем

    Векторы

    Пример 1.

    Доказать, что когда М — точка АВС и О — произвольные точки пространства, то выполняется равенство: Векторы

    Решение.

    Пусть Векторы медиана треугольника АВС. По свойствам медиан треугольника Векторы Применив к векторам Векторы и Векторы формулу вычитания векторов

    Векторы

    тогда

    Векторы

    Пример 2.

    У прямоугольного параллелепипеда рёбра Векторы, имеют длину 2, 3, 5. Вычислить длины отрезков Векторы и Векторы и угол между прямыми Векторы и Векторы.

    Решение.

    Пусть Векторы единичные вектора направленные вдоль рёбер, которые рассматриваются. Тогда Векторы (поскольку параллелепипед прямоугольный).

    рис. 9.Векторы

    Далее,

    Векторы

    Этим закончен “перевод” условия задачи на “язык” векторов.

    Теперь произведём вычисления с векторами:

    Векторы

    Наконец “переводим” полученные вектора равенства снова на “геометрический язык”. Поскольку Векторы аналогично Векторы.

    Далее поскольку Векторы, где Векторы угол между данными векторами то Векторы, отсюда получаем Векторы. Теперь с помощью тригонометрических таблиц находим значения угла Векторы.

    Разложение вектора по базису

    Базисом на площади называют упорядоченную пару неколлинеарных векторов и точку отсчёта. 

    Теорема. Любой вектор Векторы на плоскости можно разложить по двум неколлинеарным векторам Векторы и Векторы, то есть представить в виде: Векторы.

    Доказательство.

    Векторы

    Пусть векторы Векторы компланарные и векторы Векторы и Векторы неколлинеарные. От точки О отложим все три вектора и на продолжении векторов Векторы и Векторы построим параллелограмм  ONCM так, чтобы вектор Векторы был его диагональю.

    Тогда по правилу параллелограмма Векторы.

    Но Векторы, как коллинеарные векторы. Следовательно, векторВекторы.

    Числа, которые стоят при базисных векторах в разложении вектора за двумя неколлинеарными векторами называют координатами вектора в данном базисе и обозначают Векторы.

    Соответственно в пространстве базисом называется упорядоченная тройка некомпланарных векторов и точки отсчёта.  Для четырёх некомпланарных векторов справедлива следующая теорема.

    Теорема. Любой вектор  Векторы в пространстве можно разложить по трём некомпланарным векторам ВекторыВекторы и Векторы, то есть представить в виде: Векторы.

    Доказательство.

    От точки О отложим векторы  Векторы и на продолжении векторов Векторы построим параллелограмм Векторы 

    Векторы

    в котором вектор Векторы является диагональю. Как видим

    Векторы

    Числа х,у,z которые стоят при базисных векторах в разложении вектора по трём некомпланарным векторам называют координатами вектора в пространстве и обозначают Векторы. Если базисные вектора взаимно перпендикулярны (их обозначают Векторы), то вместе с точкой отсчёта они образуют декартовую систему координат, а координаты вектора в таком базисе называют декартовыми координатами. В декартовой системе координат разложение вектора будет иметь вид Векторы. Если началом вектора Векторы является точка Векторы, а концом — точка Векторы, то координаты вектора Векторы вычисляют как разность соответствующих координат точек А и В,

    Векторы

    Отсюда легко установить длину вектора как расстояние между двумя точками:

    Векторы

    Действия над векторами, заданными своими координатами

    1. При сложении двух, или более векторов их соответствующие координаты складываются:

    Векторы

    Действительно:

    Векторы

    2. При вычитании векторов соответствующие координаты вычитаются:

    Векторы

    Доказательство аналогично предыдущему.

    3. При умножении вектора на число все координаты умножаются на это число.

    Правда, для вектора Векторы и числа Векторы имеем:

    Векторы

    4. Скалярное произведение двух векторов Векторы равно сумме произведений соответствующих координат:Векторы

    Правда:

    Векторы

    Поскольку Векторы выполняется ВекторыСледовательно, мы можем записать

    Векторы

    5. Векторное произведение векторов Векторы заданных своими координатами вычисляется так:

    Векторы

    6. Смешенное произведение трёх векторов Векторы равняется:

    Векторы

    Пример 1.

    Зная координаты векторов Векторы, найти координаты векторов Векторы.

    Решение:

    Векторы

    Ответ: Векторы.

    Пример 2.

    Зная координаты векторов Векторы вычислить координаты вектора Векторы.

    Решение.

    Векторы

    Ответ: Векторы.

    Пример 3.

    Зная координаты векторов Векторы вычислить:

    а) скалярное произведение векторов Векторы

    б) векторное произведение векторов Векторы

    в) смешенное произведение векторов Векторы.

    Решение.

    Векторы

    Ответ: Векторы

    На основании приведённых выше формул действий над векторами можно установить следующие условия и соотношения для нулевых векторов

    Векторы

    1. Угол между векторами.

    Векторы

    2. Условие перпендикулярности двух векторов:

    Векторы

    (векторы перпендикулярны тогда и только тогда, когда их скалярное произведение равно нулю).

    3. Условие коллинеарности двух векторов: Векторы (векторы коллинеарные тогда и только тогда, когда соответствующие их координаты пропорциональны).

    4. Условие компланарности трёх векторов.

     Векторы

    (три вектора компланарны тогда и только тогда, когда их смешенное произведение равно нулю).

    5. Деление отрезка АВ в заданном отношении.

    Если точка Векторы делит отрезок АВ в отношении Векторы, то координаты точки М находят по формуле:

    Векторы

    Если точка М делит отрезок АВ на пополам то Векторы, и координаты точки находят согласно формуле:

    Векторы

    Действия над векторами (теория)

    а) Произведение вектора на число.
    Определение 1. Произведением вектора Векторы на число λ называется вектор Векторы,
    который имеет длину Векторы  и направление его совпадает с направлением вектора Векторыесли λ > 0,  и противоположно ему, если λ < 0 (рис.12).

    Векторы
    Рис. 12.

    Условие Векторы                                                                           (2.6)
    является условием коллинеарности двух векторов.

    б) Сложение векторов.

    Определение 2. Суммой двух векторов Векторы  и  Векторы  называется вектор   Векторы , начало которого совпадает с началом вектора Векторы,  а конец совпадает с концом вектора Векторы, при условии, что начало вектора Векторы  совпадает с концом вектора  Векторы  (правило треугольника)  (рис.13).

    Векторы

    Рис. 13.

    Понятно, что вектор Векторы в этом случае является диагональю параллелограмма, построенного на векторах Векторы  и  Векторы  (правило параллелограмма) (рис.13).
    Для векторной суммы справедливый переместительный закон
    Векторы
    Легко убедиться, что для векторной суммы имеет место соединительный
    закон  Векторы .
    Исходя из определения 2, легко находим сумму, например, четырех векторов Векторы (рис. 14).
    Векторы
    Рис. 14.
    Вектор Векторы соединяет начало первого вектора   Векторы с концом вектора  Векторы  (правило многоугольника).

    в) Вычитание векторов.
    Действие вычитание векторов можно рассматривать как обратное действие относительно сложения векторов.

    Определение. Разностью Векторы  называется вектор Векторы , который в сумме с вектором Векторы дает вектор  Векторы  (рис. 15), т.е. Векторы

    Векторы
    Рис. 15.

    Как видно из рис. 15,  одна диагональ Векторы является суммой  Векторы ,  а  вторая диагональ Векторы  является разностью векторов  Векторы и  Векторы.
    Дадим еще одно определение разности векторов.

    Определение. Разностью двух векторов Векторы и  Векторы , которые имеют общее начало, называется вектор Векторы , который соединяет концы этих векторов и направлен в сторону уменьшаемого.

    Проекция вектора на ось

    Пусть имеем произвольную ось l на плоскости и некоторый вектор Векторы (рис. 16).
    Векторы

    Рис. 16.

    Опустим из начала A вектора и из конца B перпендикуляры на ось l. Основаниями перпендикуляров будут точки A1 и B1, которые называются проекциями точек A и B.

    Величина A1B1 называется проекцией вектора Векторы на ось l и обозначается  Векторы, то есть Векторы.
    Определение 1. Проекцией вектора Векторы  на ось l называется величина отрезка  A1B1, взята со знаком плюс, если направление отрезка A1B1  совпадает с направлением оси l, и с знаком минус, если направления противоположные.

    Из точки A проведем прямую, параллельную оси l, которая пересечет отрезок  BB1 в точке C. Вектор Векторы образует с осью l угол φ. Величина отрезка AC равна величине отрезка  A1B1, а тогда из Δ ABC находим  
    Векторы    или       Векторы                                        (2.7)

    Определение 2. Проекция вектора на любую ось равна произведению длины этого вектора на косинус угла между осью и вектором.

    Если угол φ острый, то проекция  Векторы — положительное число, а если угол φ тупой, то проекция Векторы  —  отрицательное число.

    Свойства проекций.

    1. Если векторы  Векторы и  Векторы равны, то величины их проекций на одну и ту же ось l также равны, то есть:  Векторы.
    2. Проекция суммы векторов на любую ось равна сумме проекций слагаемых на ту же ось, то есть:
    Векторы

    3. Проекция разности двух векторов на ось l равна разности величин проекций на ту же ось, то есть:
    Векторы

    4. Если вектор Векторы умножен на любое число λ, то величина проекции вектора Векторы на ось также умножится на число λ, то есть: 
    Векторы
     

    Проекции вектора на оси координат

    Рассматривается прямоугольная система координат Oxyz в пространстве и произвольный вектор Векторы.
    Пусть Векторы  Векторы
    Проекции x, y, z вектора Векторы  на координатные оси называют координатами вектора и записывают Векторы.
    Если заданы две точки A (x1; y1; z1и B (x2; y2; z2), то координаты вектора Векторы находятся по формулам
    x = x2 – x1,   y = y2 –  y1,  z = z2 – z.

    Векторы

    Рис. 17

    Действительно, проведем через точки A и B плоскости, перпендикулярные оси Ox и обозначим точки их пересечения соответственно A1 и B1 (рис.17). Точки A1 и B1 имеют на оси Ox координаты   x1  и  x, но Векторы на основе формулы (2.1), а потому
    x = x2 – x1 . Аналогично доказывается, что y = y2 –  y1,  z = z2 – z.
     

    Направляющие косинусы вектора

    Пусть имеем вектор Векторы  и будем считать, что он выходит из начала координат и не находится ни в одной координатной плоскости.

    Векторы

    Рис. 18

    Через точку M проведем плоскости, перпендикулярные к осям координат, и вместе с координатными плоскостями они образуют параллелепипед, диагональ которого — отрезок OM (рис.18). Через α, β, γ обозначим углы, которые образует вектор Векторы с осями координат. Величины cos α, cos β, cos γ называются направляющими косинусами вектора Векторы. Координаты вектора Векторы.

    Квадрат диагонали прямоугольного параллелепипеда равна сумме квадратов длин трех его измерений.
    Поэтому
    Векторы или  Векторы
    Векторы                                                                     (2.8)
    Формула (2.8) выражает длину вектора через его координаты. Тогда на основе формул (2.7) и (2.8) получим
    Векторы
    Отсюда для направляющих косинусов получаем

    Векторы                  (2.9)

    Для направляющих косинусов справедливо равенство Векторы  (это вытекает из (2.9)).

    Разложение вектора по ортам

    Рассмотрим прямоугольную систему координат в пространстве и вектор, начало которого в точке O (рис.19) .

    Векторы

    Рис. 19.

    Обозначим орты осей координат Ox, Oy, Oz соответственно через  Векторы,  причем
    Векторы

    Спроецируем вектор Векторы  на координатные оси (через точку M проведем плоскости, перпендикулярные координатным осям). Проекциями точки M на координатные оси будут соответственно точки А, В, С (рис.19).

    Из прямоугольника ODMC видно, что вектор  Векторы, но из прямоугольника AOBD получаем, что вектор  Векторы.
    Тогда
    Векторы                                                                          (2.10)
    Вектор  Векторы, который соединяет точку O с точкой M (x, y, z) называется радиусом-вектором этой точки.
    Векторы Векторы называются составными или компонентами вектора Векторы, а их величины OA = x, OB = y, OC = z  координатами этого вектора. Компоненты вектора Векторывыразим через его координаты и единичные векторы Векторы, а именно Векторы.
    Подставляя эти значения в равенство (2.10), учитывая, что  Векторы, получим
    Векторы                                                                                 (2.11)

    Слагаемые  Векторы являются составными или компонентами вектора  Векторы.
    Тройка векторов  Векторы  называется координатным базисом, а разложение (2.11) называется разложением вектора по базису Векторы.  Это основная формула векторной алгебры.

    Пример 1. Построить вектор Векторы.
    Векторы

    Рис. 20.

    Решение. Компоненты вектора  Векторы  являются  Векторы  и  Векторы, и им 
    соответствует прямоугольный параллелепипед, диагональ которого является искомый вектор (рис. 20).

    Действия над векторами, заданными в координатной форме

    Если векторы заданы в координатной форме, то действия сложения, вычитания, умножения вектора на число можно заменить простыми арифметическими операциями над координатами этих векторов по таким правилам.

    Правило 1. При сложении векторов их одноименные координаты складываются

    Пусть имеем векторы Векторы и  Векторы. Найдем  Векторы.  Запишем разложение векторов  Векторы  и  Векторы.  Тогда  Векторы.
    Сложив эти равенства, получим
    Векторы.
    Итак, координаты вектора   Векторы  будут  Векторы

    Правило 2. Чтобы отнять от вектора Векторы   вектор Векторы нужно вычесть из координат вектора Векторы  соответствующие координаты вектора  Векторы, то есть
    Векторы

    Правило 3. Чтобы умножить вектор  Векторы на число λ,  нужно каждую из его координат умножить на это число. То есть, если
    Векторы   то  Векторы.
    Пример 1. Найти вектор Векторы , если   Векторы
    Решение. Выполним действия последовательно и найдем
    Векторы
    Векторы.
    Значит, Векторы

    Вектор – основные определения

    Определение вектора в пространстве ничем не отличается от определения вектора на плоскости.

    Определение 1. Вектором называется направленный отрезок, т.е. отрезок, для которого указано, какая из его граничных точек является началом, а какая — концом.

    Так же как и на плоскости, векторы обозначаются Векторы и т. п. и на чертеже изображаются стрелкой.

    Определение 2. Длиной (или модулем) вектора Векторы называется длина отрезка Векторы а направление, определяемое лучом Векторы называется направлением вектора Векторы

    Длина вектора Векторы обозначается Векторы длина вектора Векторы обозначается Векторы

    Любая точка пространства также считается вектором, который называется нулевым. Начало такого вектора совпадает с его концом, а длина равна нулю. Обозначения нулевого вектора: Векторы

    По этой ссылке вы найдёте полный курс лекций по высшей математике:

    Определение 3. Векторы Векторы и Векторы называются коллинеарными, если они лежат на одной прямой или на параллельных прямых.

    Если ненулевые векторы Векторы и Векторы лежат на параллельных прямых (следовательно, в одной плоскости), причём лучи Векторы лежат в одной полуплоскости, границей которой является прямая Векторы то векторы Векторы и Векторы называются сонаправленными в случае же, когда эти векторы принадлежат одной прямой, они называются сонаправленными, если один из лучей Векторы или Векторы целиком содержится в другом. Нулевой вектор будем считать сонаправленным с любым вектором в пространстве.

    Ясно, что сонаправленные векторы, в силу их определения, коллинеарны. Если два коллинеарных вектора не сонаправлены, то они называются противоположно направленными. Обозначения остаются обычными: Векторы (векторы Векторы и Векторы сонаправлены), Векторы (векторы Векторы и Векторы противоположно направлены).

    Определение 4. Векторы Векторы и Векторы называются равными, если Векторы и Векторы (т.е. если векторы сонаправлены и их длины равны).

    Теорема 1. От любой тонки пространства можно отложить вектор, равный данному, и притом только один.

    Доказательство этой теоремы аналогично доказательству соответствующей планиметрической теоремы.

    Возможно вам будут полезны данные страницы:

    Операции над векторами и их свойства

    Операции над векторами в пространстве аналогичны соответствующим операциям на плоскости.

    Пусть даны два вектора Векторы и Векторы В силу теоремы 1 от произвольной точки Векторы пространства можно отложить вектор Векторы а от точки Векторы — вектор Векторы Тогда вектор Векторы называется по определению суммой векторов Векторы и Векторы а описанное правило построения суммы двух векторов — правилом треугольника (рис. 1).

    Теорема 2. Сумма Векторы векторов Векторы и Векторы не зависит от выбора точки Векторы от которой при сложении откладывается вектор Векторы (Докажите эту теорему самостоятельно.)

    Правило треугольника можно сформулировать и так: для любых трёх точек Векторы пространства выполняется равенство

    Векторы

    Кроме того, сумму двух неколлинеарных векторов с общим началом можно построить и по правилу параллелограмма: Векторы где Векторы — вектор, модуль которого_равен длине диагонали параллелограмма, построенного на векторах Векторы причём вектор Векторы откладывают от той же точки, что и векторы Векторы (рис. 2).

    Все свойства операции сложения векторов, справедливые на плоскости, остаются справедливыми и в пространстве:

    1) Векторы

    2) Векторы — коммутативность (переместительный закон);

    3) Векторы — ассоциативность (сочетательный закон).

    Здесь Векторы — произвольные векторы в пространстве.

    Определение 5. Два ненулевых вектора называются противоположными, если их длины равны и эти векторы противоположно направлены.

    Вектор, противоположный данному ненулевому вектору Векторы обозначается Векторы

    Определение 6. Разностью двух векторов Векторы и Векторы называется вектор Векторы такой, что его сумма с вектором Векторы равна вектору Векторы

    Разность векторов Векторы и Векторы обозначается Векторы Таким образом, по определению Векторы если Векторы

    Разность векторов Векторы и Векторы можно найти по формуле Векторы (рис. 3) (докажите эту формулу самостоятельно). Векторы Замечание. Так же как и на плоскости, для сложения нескольких векторов в пространстве можно использовать правило многоугольника (рис. 4), только в последнем случае этот многоугольник будет пространственным (т.е. не все векторы, его составляющие, лежат в одной плоскости).

    Векторы

    Из законов сложения векторов следует, что сумма нескольких векторов не зависит от порядка слагаемых.

    Умножение (произведение) вектора на число и его свойства, так же как и свойства операции сложения, не претерпевают изменений и в пространстве.

    Определение 7. Произведением ненулевого вектора Векторы на действительное число Векторы называется вектор Векторы длина которого равна произведению длины вектора Векторы на модуль числа Векторы причём вектор Векторы сонаправлен с вектором Векторы при Векторы и противоположно направлен вектору Векторы при Векторы

    Таким образом, по определению, Векторы если Векторы причём Векторы при Векторы Ясно, что векторы Векторы коллинеарны. Если же Векторы или Векторы то Векторы

    Свойства умножения вектора на число не отличаются от аналогичных свойств на плоскости:

    1.  Векторы — ассоциативность (сочетательный закон);
    2.  Векторы —дистрибутивность относительно сложения векторов (1-й распределительный закон);
    3.  Векторы — дистрибутивность относительно сложения чисел (2-й распределительный закон).

    Здесь Векторы и Векторы — произвольные векторы, Векторы — произвольные действительные числа.

    Справедлива также и лемма о коллинеарных векторах: если векторы Векторы и Векторы коллинеарны и Векторы то существует такое действительное число Векторы

    что Векторы (ясно, что Векторы если Векторы

    Сформулируем и докажем ещё одну важную для решения некоторых задач теорему.

    Теорема 3. Пусть Векторы где Векторы — некоторое действительное число, отличное от -1, тогда точки ВекторыВекторы принадлежат одной прямой. Для произвольной точки Векторы пространства справедливо равенство:

    Векторы

    Доказательство 

    1. Из равенства Векторы следует, что векторы Векторы коллинеарны, и так как Векторы — общая точка прямых Векторы и Векторы эти прямые совпадают, поэтому точки Векторы принадлежат одной прямой.

    2. Пусть Векторы — произвольная точка пространства. Тогда Векторы и поскольку ВекторыВекторы откуда Векторы Поделив обе части последнего равенства на Векторы приходим к формуле (1). Теорема доказана.

    З. Компланарные и некомпланарные векторы

    Следующее понятие уже не имеет аналога в планиметрии.

    Определение 8. Векторы называются компланарными, если лучи, задающие их направления, параллельны некоторой плоскости.

    Замечание. Из определения 8 следует, что при откладывании от одной точки векторов, равных нескольким данным компланарным векторам, получим векторы, лежащие в одной плоскости. Таким образом, компланарные векторы лежат либо в одной плоскости, либо в параллельных плоскостях.

    Очевидно, что любые два вектора компланарны и любые три вектора, два из которых коллинеарны, также являются компланарными (поясните). Рассмотрим теперь условия, при которых три вектора, из которых никакие два не коллинеарны, являются компланарными.

    Теорема 4. Векторы Векторы из которых никакие два не коллинеарны, являются компланарными в том и только том случае, если существуют такие действительные числа Векторы и Векторы что

    Векторы (иными словами, векторы Векторы являются компланарными в том и только том случае, если один из них можно выразить через два других, или, как говорят, разложить по двум другим).

    Доказательство

    1. Пусть векторы Векторы компланарны. Докажем, что для них имеет место равенство (5). Отложим от произвольной

    точки Векторы векторы ВекторыВекторы Векторы Векторы лежат в одной плоскости (см. замечание). Проведём через точку Векторы прямую Векторы до пересечения с прямой Векторы в точке Векторы и прямую Векторы до пересечения с прямой Векторы в точке Векторы (см. рис. 8). Так как векторы Векторы коллинеарны, по лемме о коллинеарных векторах (см. §1.2) существуют такие действительные числа Векторы и Векторы что Векторы ВекторыНо по правилу параллелограмма Векторы откуда Векторы Обратно, пусть выполнено равенство (5).

    Докажем, что векторы Векторы компланарны. Векторы Векторы при откладывании от одной точки определяют некоторую плоскость. Согласно правилу параллелограмма и равенству (5) вектор Векторы принадлежит той же плоскости, откуда следует, что векторы Векторы Векторы и Векторы а значит, и векторы Векторы компланарны. Теорема доказана.

    Отложим от произвольной точки Векторы пространства векторы Векторы Векторыгде Векторы — три данных некомпланарных вектора, и рассмотрим параллелепипед Векторы построенный на векторах Векторы (рис. 9). Тогда сумму векторов Векторыможно найти следующим образом: ВекторыВекторы Это правило сложения трёх некомпланарных векторов называется правилом параллелепипеда.

    Если векторы Векторы не являются компланарными и для вектора Векторы имеет место равенство Векторы где Векторы — некоторые действительные числа, то говорят, что вектор Векторы разложен по трём некомпланарным векторам

    Векторы а числа Векторы называются коэффициентами разложения.

    Следующая теорема, называемая теоремой о разложении вектора по трём некомпланарным векторам, является основной во всей элементарной (школьной) векторной алгебре.

    Теорема 5. Любой вектор Векторы пространства можно разложить по трём данным некомпланарным векторам Векторы причём коэффициенты разложения определятся единственным образом. Доказательство. 1. Если векторы Векторы и Векторы коллинеарны, то ВекторыВекторы и теорема доказана.

    2. Пусть векторы Векторы и Векторы не коллинеарны. Отложим от произвольной точки Векторы пространства векторы ВекторыВекторы (рис. 10). Проведём через точку Векторы прямую Векторы до пересечения с плоскостью Векторы в точке Векторы Через точку Векторы в плоскости Векторы проведём прямую Векторы до пересечения с прямой Векторы в точке Векторы (в частности, если Векторы то точка Векторы совпадает с точкой Векторы Согласно правилу многоугольника Векторы но векторы Векторы Векторы по построению коллинеарны, поэтому в силу леммы о коллинеарных векторах ВекторыВекторы где Векторы — некоторые действительные числа Таким образом, учитывая, что Векторы приходим к равенству ВекторыВекторы

    3. Докажем теперь, что разложение вектора Векторы по данным векторам Векторы единственно. Допустим, что это не так, т.е. существует ещё одно разложение Векторы в котором хотя бы один коэффициент не равен соответствующему коэффициенту в полученном нами разложении. Пусть, например, Векторы Вычтем последнее равенство из предпоследнего.

    Тогда Векторы отсюда ВекторыВекторы– т. е. векторы Векторы компланарны, что противоречит условию теоремы. Значит, наше допущение о ещё одном разложении неверно, т.е. разложение вектора Векторы по данным векторам Векторы единственно. Теорема доказана.

    Итак, любой вектор Векторы пространства можно разложить по трём данным некомпланарным векторам Векторы причём единственным образом. Заданную тройку некомпланарных векторов Векторы называют базисом, сами векторы Векторы — базисными векторами, а разложение вектора Векторы по векторам Векторы называют разложением по данному базису Векторы

    Координаты вектора

    Так же как и на плоскости, в пространстве помимо координат точки вводятся координаты вектора. Рассмотрим три попарно перпендикулярных вектора Векторы отложенных от некоторой точки Векторы пространства, таких, что Векторы (например, их можно направить по рёбрам единичного куба). Эти векторы, очевидно, не являются компланарными. Поэтому, в силу теоремы 5, любой вектор Векторы можно разложить_по векторам Векторы причём единственным образом: Векторы Введём прямоугольную систему координат с началом в точке Векторы так, чтобы направления осей Векторы совпали_с направлениями векторов Векторы соответственно. Тогда векторы Векторы называются единичными векторами осей координат, а числа Векторы — координатами вектора Векторы в системе координат Векторы (обозначения: Векторы

    Свойства векторов пространства, заданных своими координатами, аналогичны соответствующим свойствам векторов на плоскости:

    1. Два вектора равны в том и только том случае, если равны их координаты.
    2. Координаты суммы (разности) двух векторов равны суммам (разностям) соответствующих координат этих векторов, т.е. для векторов Векторы получаем Векторы
    3. При умножении вектора на число каждая его координата умножается на это число, т.е. для вектора Векторы и действительного числа Векторы получаем Векторы

    Докажем, например, свойство 2. Так как ВекторыВекторы то, согласно свойствам сложения векторов и умножения вектора на число, Векторы т. е. вектор Векторы имеет координаты Векторы что и требовалось доказать. Остальные свойства доказываются аналогично.

    Скалярное произведение векторов и его свойства

    Определение скалярного произведения векторов Векторы и Векторы в пространстве ничем не отличается от аналогичного определения для векторов на плоскости.

    Определение 11. Скалярным произведением векторов Векторы называется произведение длин этих векторов на косинус угла между ними (обозначение: Векторы Таким образом, по определению,

    Векторы

    Теорема 8. Два ненулевых вектора Векторы взаимно перпендикулярны тогда и только тогда, когда их скалярное произведение равно нулю, т. е.

    Векторы

    Доказательство этой теоремы вытекает из формулы (9).

    Определение 12. Скалярным квадратом вектора Векторы называется скалярное произведение Векторы Скалярный квадрат обозначается Векторы т.е. по определению Векторы

    Так как Векторы то

    Векторы

    Таким образом, длина вектора равна квадратному корню из его скалярного квадрата.

    Замечание. Скалярное произведение есть число, поэтому грубой ошибкой явилась бы запись: Векторы

    Если векторы Векторы и Векторы заданы своими координатами: ВекторыВекторы то скалярное произведение может быть выражено через их координаты.

    Теорема 9. Скалярное произведение векторов равно сумме произведений их соответственных координат, т. е.

    Векторы

    Доказательство. Отложим от произвольной точки Векторы пространства векторы Векторы При этом, как мы знаем, соответствующие координаты векторов Векторы и Векторы а также Векторы и Векторы будут равны, а угол Векторы По теореме косинусов для треугольника Векторы получим

    Векторы

    итак как Векторы имеем ВекторыВекторы откуда Векторы Но

    Векторы

    поэтому

    Векторы

    Решение любой геометрической задачи на вычисление сводится, в сущности, к нахождению величин двух типов: расстояний и углов. Если в пространстве задан некоторый базис (в частности, прямоугольный), т. е. тройка некомпланарных векторов, то на основании теоремы 5 любой вектор пространства можно разложить по векторам этого базиса, причём единственным образом.

    Если известны длины векторов, образующих базис, углы между ними и разложение некоторого вектора по векторам этого базиса, то, используя свойства скалярного произведения, можно определить длину такого вектора и угол, образуемый им с любым другим вектором, разложение которого по векторам этого базиса известно.

    Таким образом, векторы позволяют находить решения довольно широкого класса геометрических задач, а умение определять разложение вектора по базисным векторам является важнейшим фактором их решения.

    Для решения задач о разложении вектора по трём данным некомпланарным векторам, разумеется, необходимо, помимо теоремы 5, знание предшествующего ей материала.

    Примеры с решением

    Задача 1.

    Основанием четырёхугольной пирамиды Векторы является параллелограмм Векторы Точки Векторы и Векторы — середины рёбер Векторы и Векторы соответственно. Найдите разложение векторов Векторы по векторам Векторы

    Решение (см. рис. 14).

    1. Векторы но Векторы поэтому Векторы

    2. Так как Векторы — середина Векторы но ВекторыВекторы (см. следствие 1 теоремы 3), поэтому ВекторыВекторы

    Ответ: Векторы

    Заметим, что в разложении вектора Векторы по векторам Векторы коэффициент разложения при векторе Векторы равен нулю, а это означает, в силу теоремы 4, что векторы Векторы компланарны. Если заранее «увидеть», что Векторы где Векторы — середина Векторы (отсюда Векторы то разложение вектора Векторы можно было бы найти проще. Но векторный метод тем и хорош, что, даже не обладая развитым пространственным воображением, а лишь зная основные определения и теоремы, можно получить правильный ответ (пусть и не всегда самым оптимальным путём)!

    Задача 2.

    Пусть Векторы — точка пересечения медиан треугольника Векторы — произвольная точка пространства. Найдите разложение вектора Векторы по векторам Векторы

    Решение (см. рис. 15). Пусть Векторы — середина ребра Векторы Так как Векторы — точка пересечения медиан треугольника Векторы точки Векторы принадлежат одной прямой, причём, в силу теоремы о точке пересечения медиан треугольника, ВекторыСогласно следствию I теоремы 3 Векторы Тогда Векторы

    Векторы

    Ответ: Векторы

    Векторы и их решение

    Вектором называется направленный отрезок. Направление отрезка показывается стрелкой. Различают начало и конец отрезка. 

    Два вектора называются равными между собой, если каждый из них можно получить параллельными перенесениями другого. 

    Равные векторы являются параллельными (колинеарными), имеют одно и то же направление и одинаковую длину. Длина вектора Векторы называется абсолютной величиной или модулем вектора и обозначается Векторы

    Вектор называется нулевым (ноль- вектором), если он имеет нулевую длину, то есть его конец сходится с началом. 

    Чтобы найти сумму двух векторов Векторы и Векторы совместим начало вектора Векторы с концом вектора Векторы.

    Суммой Векторы векторов Векторы и Векторы  называется вектор, начало которого сходится с началом вектора Векторы, а конец – с концом вектора Векторы (рис. 1.1).

    Векторы Правило треугольника

    Векторы Правило параллелограмма 

    Векторы

    Для складывания векторов имеют место такие законы: 

    1) переставной (коммутативный)

    Векторы

    2) связующий 

    Векторы

    3) для каждого вектора Векторы существует противоположный Векторы такой, что 

    Векторы

    4)Векторы

    5) для некоторых двух  векторов Векторы и Векторы  выполняются неравенства: 

    Векторы

    Если вектор Векторы образует угол Векторы с осью Векторы (рис. 1.2), то проекцию вектора Векторы на ость называется величина 

    Векторы

    Пусть вектор имеет начало в точке Векторы а конец – в точке  Векторы Тогда величины Векторы Векторы являются проекциями вектора Векторы на оси Векторы Проекции вектора однозначно определяют вектор. Потому имеет место равенство 

    Векторы

    Если вектор Векторы то проекция суммы векторов 

    Векторы

    Произведением вектора Векторы на число Векторы называется вектор Векторы длина которого равна Векторы Умножение вектора на число имеет свойство ассоциативности и дистрибутивности, то есть для произвольных чисел Векторы и векторов Векторы и Векторы справедливы равенства: 

    Векторы

    Любой вектор Векторы можно записать в видеВекторы

    где Векторы – единичные векторы, Векторы Векторы называются компонентами вектора   Векторы  (рис. 1.3) .

    Векторы

    Векторы

    Пример 1.73  

    Даны два вектора: Векторы и Векторы 

    Найти вектор Векторы

    Решение Векторы

    Признаком колинеарности двух векторов Векторы  и  Векторы  является пропорциональность их координат: 

    Векторы

    Скалярным произведением двух векторов Векторы  и  Векторы  называется число Векторы которое равно произведению их модулей на косинус угла между ними: 

    Векторы

    Скалярное произведение можно записать в таком виде: 

    Векторы

    Если векторы Векторы  и  Векторы  заданы своими координатами, то их скалярное произведение вычисляется по формуле: 

    Векторы

    Учитывая формулы (1.18) и (1.19), можно найти косинус угла между векторами  Векторы  и  Векторы

    Векторы

    Отсюда получается условие перпендикулярности двух векторов: если Векторы  и Векторы   или в координатной форме: 

    Векторы

    Среди свойств скалярного произведения отметим так: 

    Векторы

    Векторным произведением вектора Векторы на вектор Векторы называется вектор Векторы который имеет такие свойства: 

    1) длина вектора Векторы равна произведению длин сомножителей на синус угла между ними: Векторы

    2) вектор Векторы перпендикулярный к векторам Векторы и Векторы

    3) из конца вектора Векторы  кратчайший поворот от Векторы  к  Векторы  является таким, что происходит против часовой стрелки (рис. 1.4). 

    Векторы

    Заметим, что Векторы а модуль векторного произведения равен плоскости параллелограмма, построенного на векторах Векторы  и   Векторы, если у них общее начало.  

    В координатной форме векторное произведение векторов Векторы и Векторы можно записать в виде:  

    Векторы

    Смешанным или скалярно – векторным произведением трех векторов Векторы называется векторное произведение векторов  Векторы  и   Векторы, скалярно умноженный на вектор Векторы то есть Векторы

    Если векторы Векторы – компланарны, то есть расположены в одной плоскости или на параллельных плоскостях, то их смешанное произведение равно нулю. 

    Если известные координаты сомножителей ВекторыВекторы то смешанное произведение вычисляется по формуле: 

    Векторы

    Если три ненулевых Векторы разложены в одной плоскости (компланарны), то из смешанное произведение Векторы

    Следует, в координатной форме условие компланарности трех ненулевых векторов имеет вид: 

    Векторы

    Решение примеров:

    Пример 1.74 

    Заданы координатами точек Векторы Векторы и Векторы Найти: 

    1) вектор Векторы если Векторы

    2) угол между векторами Векторы и Векторы

    3) координаты вектора Векторы

    4) объем пирамиды с вершинами в точках Векторы

    Решение 

    1) По формуле (1.14) находим 

    Векторы

    тогда Векторы

    2) Косинус угла между векторами Векторы и Векторы вычислим по формуле (1.20): 

    Векторы

    Поскольку косинус угла отрицательный, то угол Векторы тупой. 

    3) Координаты векторного произведения находим по формуле (1.22):

    Векторы

    Векторы

    4) Чтобы найти объем пирамиды, найдем сначала смешанное произведение векторов, что выходят из одной вершины пирамиды: 

    Векторы

    Тогда объем пирамиды

    Векторы

    Собственные числа и собственные векторы

    Вектор – столбец Векторы  называется собственным вектором квадратной матрицы Векторы Векторы – ого порядка, что соответствует собственному значению Векторы если он удовлетворяют матричному уравнению Векторы или ВекторыВекторы

    Тут Векторы – единичная матрица Векторы – ого порядка, а Векторы – нулевой вектор – столбец. При условии, что Векторы получим характеристическое уравнение для определения собственных значений Векторы

    Векторы

    Координаты собственного вектора Векторы что соответствуют собственному значению Векторы является решением системы уравнений: 

    Векторы

    Собственный вектор обозначаются с точностью к постоянному множителю.

    Решение примеров:

    Пример 1.90.

    Обозначить  собственные определения и собственные векторы матрицы

    Векторы

    Решение. Характеристические уравнения данной матрицы имеет вид (1.24): 

    Векторы или Векторы

    отсюда получается, что матрица Векторы имеет два собственных значения Векторы и Векторы Собственный вектор Векторы что соответствует Векторы обозначаются с системой уравнений вида (1.25)

    Векторы  или Векторы

    которое приводится к одному уравнению Векторы

    Возьмем Векторы получим решение в виде Векторы

    Следует, первый собственный вектор является 

    Векторы

    Второй вектор Векторы что соответствует собственному значению Векторы определяется из системы уравнений вида (1.25)

    Векторы

    Эта система уравнений так же приводится к одному уравнению Векторы положив Векторы запишем ее решение в виде Векторы Следует, второй собственный вектор: 

    Векторы

    Таким образом, матрица Векторы имеет два разных определения Векторы и Векторы и два собственных вектора, равных Векторы и Векторы (с точностью к постоянному множителю). 

    Пример 1.91 

    Найти собственные векторы и собственные значения матрицы 

    Векторы

    Решение. Характеристическое уравнение

    Векторы

    Раскрыв определитель получим: 

    Векторы

    Корень Векторы – кратный, показатель кратности Векторы корень Векторы – простой, Векторы

    Система уравнений для определения собственных векторов имеет вид: 

    Векторы

    Последовательно подставим Векторы и Векторы в записанную систему: 

    Векторы

    Векторы

    Фундаментальная система уравнений получается, если свободным переменным Векторы последовательно дать значения Векторы

    Векторы

    Получили два линейно независимые собственные векторы. Вся совокупность векторов, что соответствуют собственному значению Векторы имеет вид: 

    Векторы

    Векторы

    Векторы

    Фундаментальная система решений получается, если взять Векторы

    Векторы

    Векторная алгебра

    Понятие «вектор» (от лат. vector – носитель), как отрезка, имеет определенную длину и определенное направление, впервые появилось в работах по построению числовых систем в ирландского математика Уильяма Гамильтона (1805-1865). Это понятие связано с объектами, которые характеризуются величиной и направлением, например, скорость, сила, ускорение. При этом скорость можно понимать в широком смысле: скорость изменения издержек производства, доходов, спроса, потребления и предложения и др. Вектор может указывать направление наибольшего возрастания или убывания функции, описывающей различные экономические процессы. Векторы, рассмотренные в данном разделе, является частным случаем Векторы-мерных векторов: они предполагают геометрическую интерпретацию, потому что принадлежат к векторным линейных пространств размерности Векторы

    Для графического изображения решения экономических задач на плоскости и в пространстве применяются средства аналитической геометрии. Аналитическая геометрия – математическая наука, объектом изучения которой являются геометрические фигуры, а предметом – установление их свойств средствами алгебры с помощью координатного метода. Теоретической базой этой науки является частично известна из школы векторная алгебра.

    Основателем метода координат и, вместе с тем, аналитической геометрии является Рене Декарт (1596-1650) – французский философ, математик, физик и физиолог. Его именем и названа известная «декартова прямоугольная система координат», которая позволяет определить положение фигуры на плоскости и тела в пространстве.

    После изучения данной темы вы сможете:

    ● использовать инструмент векторной алгебры для геометрического изображения и анализа объектов экономических процессов;
    ● применять уравнение прямой линии на плоскости для геометрической интерпретации зависимости между функциональному признаку и аргументом, что на нее влияет;
    ● применять уравнение кривых второго порядка при построении нелинейных математических моделей экономических задач;
    ● осуществлять геометрическую интерпретацию решений экономических задач с помощью поверхностей и плоскостей.

    Векторы: основные определения, линейные операции

    Выберем на произвольной прямой (в Векторы или в Векторы) отрезок Векторы и укажем, которую из точек Векторы или Векторы считать начальной (началом отрезка), а какую – конечной (концом отрезка). Конец отрезка обозначают стрелке и говорят, что на отрезке задано направление. Отрезок Векторы с заданным на нем направлением, или коротко – направленный отрезок, называется вектором. Вектор обозначается символом Векторы или строчными буквами латинского
    алфавита с чертой: Векторы и др. (Рис. 6.1). 

    Векторы

    Рис. 6.1

    В применимых задачах естественных наук существенным является обстоятельство – где, в какой точке находится начало вектора. Например, результат действия силы зависит не только от ее величины и направления действия, но и от того, в какой точке она прикладывается.

    Вектор, для которого фиксированная (не фиксирована) начальная точка называется связанным (свободным). Векторы, которые применяются в экономических задачах, как правило, не являются связанными, поэтому в дальнейшем будем рассматривать преимущественно свободные векторы

    Длиной, или модулем, вектора называется длина соответствующего отрезка и обозначается одним из символов: Векторы

    Нулевым вектором 0, или ноль-вектором, называется вектор, длина которого равна нулю, а направление его считается произвольным (неопределенным).

    Единичным вектором Векторы называется вектор, длина которого равна единице.

    Равными векторами называются векторы, которые принадлежат одной прямой или параллельным прямым, одинаково направлены и имеют равные длины.

    Взаимно противоположными называются векторы, которые принадлежат одной прямой или параллельным прямым, имеют равные длины, но противоположно направлены. Вектор, противоположный вектору Векторы, обозначают символом Векторы.

    Коллинеарными называют векторы, которые принадлежат одной прямой или параллельным прямым.

    Компланарными называются векторы, которые принадлежат одной плоскости или параллельным плоскостям.

    Линейные операции над векторами

    Будем считать, что векторы Векторы принадлежат одни плоскости. Осуществляя параллельный перенос одного из векторов Векторы, совместим начало вектора Векторы с концом вектора Векторы (или наоборот) и по отрезками, соответствующие векторам, как по двум сторонам, построим треугольник (рис. 6.2 а).

    1. Суммой векторов Векторы называется вектор Векторы, который определяется третьей стороной треугольника, с началом в начале вектора Векторы. Порядок построения суммы двух векторов по этому определению называют правилом треугольника.

    Параллельный перенос можно осуществить и так, что объединятся начала векторов Векторы и Векторы, тогда на векторах как на сторонах построим параллелограмм (рис. 6.2 б), и придем к известному из школьного курса алгебры правилу параллелограмма.

    Векторы

    Рис. 6.2

    Правило треугольника обобщается на произвольное конечное число векторов. Если параллельным переносом расположить векторы так, что конец предыдущего вектора (начиная с первого) является началом следующего, то результирующим будет вектор, соединяющий начало первого вектора слагаемого с концом последнего (рис. 6.3):

    Векторы

    Векторы

    Рис. 6.3

    Соответствующее правило называют правилом многоугольника.
    Свойства суммы векторов:
    1) переставная, или коммутативна:

    Векторы

    2) соединительная, или ассоциативная:

    Векторы

    3) Векторы

    4) Векторы

    Разницу Векторы можно рассматривать как сумму вектора Векторы с вектором, противоположным вектору Векторы

    Векторы

    Умножения вектора на скаляр

    Пусть Векторы – некоторое действительное число Векторы. Произведением вектора Векторы со скаляром Векторы называется вектор Векторы, модуль которого равен произведению модулей Векторы, а направление Векторы совпадает с направлением Векторы, если Векторы, или противоположно направлению Векторы, если Векторы (рис. 6.4):

    Векторы

    Векторы

    Рис. 6.4

    ПриВекторы вектор Векторы превращается в ноль-вектор Векторы.
    Свойства умножения вектора на скаляр:
    1) переставной или коммутативных закон:

     Векторы где Векторы

    2) соединительный, или ассоциативный закон:

    Векторы где Векторы

    3) распределительный или дистрибутивный закон:

    Векторы где Векторы

    4) Векторы

    5) Векторы

    Из определения умножения вектора на скаляр следует необходимое и достаточное условие коллинеарности двух векторов: вектора Векторы и Векторы коллинеарны тогда и только тогда, когда каждый из них является произведением другого из скаляром:

    Векторы

    Известно, что три ненулевые векторы Векторы и Векторы компланарны тогда и только тогда, когда один из них является линейной комбинацией двух других:

    Векторы компланарны Векторы

    Рассмотрим понятие, имеет очень важное значение в теории векторов – проекции вектора на ось (прямую, имеет направление; заданное направление считать положительным, противоположное направление – отрицательным).

    Компонентой вектора Векторы относительно оси Векторы называют вектор, начало которого является проекцией начала вектора Векторы на ось Векторы, а конец – проекцией конца вектора Векторы на ось Векторы (рис. 6.5).

    Векторы

    Рис. 6.5

    Проекцией вектора Векторы на ось Векторы называют скаляр, равный длине компоненты вектора Векторы относительно оси Векторы со знаком Векторы, если направление компоненты совпадает с направлением оси Векторы, или со знаком Векторы, если ее направление противоположно направлению оси:

    Векторы

    Основные свойства проекции вектора на ось

    1. Проекция вектора на ось Векторы равна произведению длины вектора Векторы с косинусом угла между вектором и осью:

    Векторы

    2. Проекция суммы двух векторов на эту ось равна сумме их проекций на эту ось:

    Векторы

    Это свойство обобщается на любое конечное число векторов.

    3. Проекция на ось произведения вектора со скаляром равна произведению со скаляром проекции самого вектора на ось:

    Векторы

    Прямоугольная система координат в пространстве. Координатная и алгебраическая формы задания векторов

    Пусть в трехмерном векторном пространстве Векторы задана прямоугольная декартова система координат Векторы, что определяется тремя взаимно перпендикулярными числовыми осями – осями, на которых указано масштаб (единицу длины) – с общей точкой Векторыначалом координат (рис. 6.6).

    Векторы

    Рис. 6.6

    Выберем в пространстве произвольную точку Векторы и соединим ее отрезком прямой с началом координат Векторы. Вектор Векторы, началом которого является начало координат Векторы, а концом данная точка Векторы, называется радиусом-вектором точки Векторы. Отметим, что радиусы-векторы точек пространства являются связанными векторами. 

    Под декартовыми прямоугольными координатами точки Векторы понимают проекции ее радиус-вектора Векторы на оси Векторы

    Векторы

    Точка Векторы с координатами Векторы обозначается через Векторы. Вектор Векторы каждой точки пространства (кроме точки Векторы) определяет прямоугольный параллелепипед с диагональю, что является отрезком, на котором построено вектор Векторы (рис. 6.6).

    Измерениями параллелепипеда есть модули координат точки Векторы. Длина диагонали параллелепипеда определяется по формуле: 

    Векторы

    Углы Векторы, которые образованы радиусом-вектором Векторы с координатными осями Векторы называются его направляющими углами. 

    Векторы

    откуда:

    Векторы

    Косинусы направляющих углов называются направляющими косинусами радиус-вектора Векторы. С (6.4) получаем свойства:
    1) направляющие косинусы являются координатами единичного радиус-вектора: Векторы

    2) сумма квадратов направляющих косинусов вектора Векторы равна единице: Векторы

    Понятие «координата», «направляющие углы», «направляющие косинусы» без изменений переносятся на любые свободные векторы, потому начало каждого из них параллельным переносом можно поместить в начало Векторы, дает радиус вектор определенной точки.

    Координатами любого вектора Векторы в пространстве называются его проекции на оси координат. Они обозначаются символами Векторы и пишут: Векторыили Векторы, где согласно определению координат:

    Векторы

    Задача вектора тройкой его координат Векторы, называют координатной формой задачи.

    Для единичных векторов Векторы, расположенных соответственно на осям Векторы, имеем:

    Векторы

    Длина произвольного вектора Векторы и его направляющие косинусы вычисляются по формулам:

    Векторы

    Найти длину и направляющие косинусы вектора ВекторыВекторы

    По формулам (6.5) имеем: 

    Векторы

    Установим связь между координатами вектора – числами – и его компонентами – векторами – с помощью единичных векторов Векторы (рис. 6.7).

    Векторы

    Рис. 6.7

    Компонентами вектора Векторы относительно координатных осей являются векторы Векторы Векторы (рис. 6.7). Согласно операции сложения векторов по правилу многоугольника получаем:

    Векторы

    Следовательно, любой вектор Векторы в трехмерном пространстве является суммой трех его компонент относительно координатных осей:

    Векторы

    Изображение вектора с Векторы в виде суммы произведений координат с единичными векторами (ортами) называют алгебраической формой задания вектора.

    Согласно свойствами операций над векторами, алгебраическая форма задания дает возможность установить результаты действий над векторами, заданными в координатной форме.
    1. При добавлении (вычитании) двух векторов с Векторы: Векторы и Векторы, их соответствующие по номеру координаты прилагаются (вычитаются):

    Векторы

    Действительно, по свойствам ассоциативности и дистрибутивности имеем:

    Векторы

    2. При умножении вектора Векторы на скаляр Векторы все его координаты умножаются на этот скаляр:

    Векторы

    Действительно, согласно распределительным свойствам умножения скаляра на сумму векторов имеем:

    Векторы

    Скалярное, векторное, смешанное произведения векторов

    Скалярным произведением двух векторов Векторы и Векторы называется число (скаляр), равное произведению их модулей с косинус угла между ними Векторы и обозначается Векторы:

    Векторы

    Вместо Векторы часто пишут Векторы или используют обозначения Векторы. Название этой операции согласуется с ее сути, а именно: скалярное произведение является скаляром, то есть числом.

    Для определения угла Векторы между векторами Векторы и Векторы совмещают их начала и рассматривают угол между двумя лучами Векторы и Векторы (рис. 6.8). Если угол Векторы острый, то Векторы, если тупой, то Векторы.

    Основные свойства скалярного произведения векторов вытекают из его определения (6.7).

    1. Скалярное произведение Векторы ненулевых векторов равно нулю тогда и только тогда, когда векторы взаимно перпендикулярны (ортогональные):

    Векторы

    2. Скалярный квадрат вектора равен квадрату его модуля, то есть

    Векторы

    3. Скалярное произведение подчиняется всем законам арифметики чисел относительно линейных операций:

    Векторы

    Векторы

    4. Скалярное произведение двух векторов равно произведению модуля одного из них с проекцией второго на ось, направление которого определяется первым вектором:

    Векторы

    Доказательство этого свойства основывается на определении (6.3).

    Скалярное произведение векторов Векторы и Векторы, заданных в координатной форме. Пусть имеем два вектора Векторы

    1. Вычислим скалярные произведения единичных векторов Векторы По свойству Векторы Для других пар на основании свойства 1 имеем: Векторы

    2. Находим произведение Векторы, подавая векторы в алгебраической форме (6.6) и используя распределительный закон:

    Векторы

    Раскрываем скобки и получаем:

    Векторы

    Скалярное произведение двух векторов равно сумме произведений одноименных координат. Это полностью совпадает с определением скалярного произведения Векторы-мерных векторов.

    Как следствие из (6.12) при Векторы получаем формулу (6.5) модуля вектора через его координаты:

    Векторы

    Определим угол между двумя ненулевыми векторами Векторы и Векторы, заданные в координатной форме. Воспользуемся определением скалярного произведения (6.7) и соотношения (6.5). В результате получаем:

    Векторы

    Следовательно, косинус угла между двумя векторами определяется формулой: 

    Векторы

    Отсюда Векторы

    В результате с соотношением (6.13) получим критерий ортогональности двух векторов, заданных в координатной форме: 

    Векторы

    Критерием коллинеарности векторов Векторы и Векторы, заданных в координатной форме является пропорциональность их координат:

    Векторы

    Векторное произведение двух векторов

    Пусть Векторы и Векторы – векторы пространства Векторы Векторы, определяющие некоторую плоскость Векторы. Вектор Векторы называется векторным произведением векторов Векторы и Векторы, если вектор Векторы удовлетворяет условиям: 

    1) модуль его численно равен площади параллелограмма, построенного на векторах Векторы и Векторы как на сторонах;
    2) он перпендикулярный плоскости параллелограмма Векторы и направленный так, что поворот вектора Векторы до совмещения с вектором Векторы кратчайшим путем наблюдается с конца вектора Векторы против часовой стрелки (рис. 6.9).

    Векторы

    Рис. 6.9

    Векторное произведение обозначается символами: Векторы, или Векторы

    Следовательно,

    Векторы

    где Векторынаименьший из углов Векторы что соответствует совмещению Векторы с Векторы поворотом вектора Векторы против часовой стрелки.

    Основные свойства векторного произведения вытекают из его определения.
    1. Векторное произведение ненулевых векторов равно ноль-вектору тогда и только тогда, когда векторы Векторы и Векторы коллинеарны:

    Векторы

    Еще одним критерием коллинеарности векторов является равенство нулевому вектору их векторного произведения.

    2. Векторные произведения с разным порядком сомножителей являются взаимно противоположными векторами:

    Векторы

    Это означает, что векторное произведение не подчиняется переставному (коммутативному) закону.

    3. Векторное произведение подчиняется ассоциативному закону относительно скалярного множителя и дистрибутивному закону относительно сложения:

    Векторы

    где Векторы

    Векторное произведение векторов Векторы и Векторы, заданных в координатной форме. Пусть имеем два ненулевые векторы: Векторы

    1. Определяем векторные произведения ортов Векторы (рис. 6.10).

    Векторное произведение одноименных векторов по свойству 1 дает ноль вектор:

    Векторы

    Однако все векторные произведения разноименных единичных векторов будут давать единичные векторы:

    Векторы

    Векторы

    Рис. 6.10

    Рассмотрим, например, произведение Векторы. Совмещение Векторы с Векторы кратчайшим путем (указано дугой со стрелкой на рис. 6.10) происходит против часовой стрелки, если смотреть с конца вектора Векторы, следовательно, Векторы. Тогда по свойству Векторы

    2. Находим произведение Векторы, подавая векторы в алгебраической форме и используя арифметические свойства (6.18) и соотношения (6.19):

    Векторы

    Множители при Векторы это вскрытые определители 2-го порядка, поэтому Векторы

    Коэффициенты при единичных векторах в соотношении (6.20) являются координатами вектора Векторы как векторного произведения векторов Векторы и Векторы.

    Если символы Векторы в соотношении (6.20) считать элементами первой строки определителя 3-го порядка, то окончательно получим представление Векторы в виде определителя: 

    Векторы

    Найдем векторное произведение векторов Векторы и Векторы

    Векторы

    Модуль векторного произведения Векторы определяет площадь параллелограмма, построенного на векторах Векторы и Векторы

    Смешанным произведением трех векторов Векторы и Векторы называется векторное произведение двух из них, умножен скалярно на третий вектор, то есть Векторы и т. д.

    Смешанное произведение можно обозначать тройкой векторов Векторы, в которой первые два элемента считают связанными векторным произведением, а результат векторного произведения умножают на третий вектор скалярно, то есть Векторы – это все равно, что Векторы. Понятно, что результатом смешанного произведения является скаляр, поскольку векторное произведение Векторы является вектором (обозначим его через Векторы), а произведение Векторы дает скаляр.

    Геометрическая интерпретация смешанного произведения. Пусть Векторы и Векторы – некомпланарные векторы. Построим на этих векторах как на ребрах параллелепипед (рис. 6.11).

    Векторы

    Рис. 6.11

    Вектор Векторы по длине численно равна площади параллелограмма, построенного на векторах Векторы и Векторы как на сторонах. Этот параллелограмм является основой параллелепипеда, построенного на векторах Векторы и Векторы. Вектор Векторы является перпендикулярным плоскости параллелограмма.

    Согласно (6.11) скалярное произведение Векторы можно представить как произведение модуля Векторы и проекции вектора Векторы на ось, определяется вектором Векторы:

    Векторы

    где Векторы, причем Векторы является положительным числом, если угол между векторами Векторы и Векторы острый, и отрицательным, если этот угол тупой. По модулю эта проекция равна высоте параллелепипеда Векторы.

    Модуль смешанного произведения трех векторов численно равен объему параллелепипеда Векторы, построенного на векторах как на ребрах:

    Векторы

    Основные свойства смешанного произведения вытекают из его определения и геометрической интерпретации.
    1. Смешанное произведение ненулевых векторов равно нулю, если по крайней мере два из трех векторов коллинеарны или все три – компланарны, и наоборот.

    Необходимым и достаточным условием компланарности трех ненулевых векторов является равенство нулю их смешанного произведения:

    Векторы компланарны Векторы

    Свяжем с изображенными на плоскости векторами Векторы круг (рис. 6.12). Перечисление векторов, начиная с любого, против часовой стрелки назовем положительным, или циклическим, перестановкой векторов, в противном случае – отрицательной перестановкой.

    2. Циклическая перестановка трех сомножителей смешанного произведения не меняет его величины, а отрицательное перестановки меняет его знак на противоположный:

    Векторы

    Смешанное произведение векторов, заданных в координатной форме

    Пусть имеем три ненулевые векторы Векторы По определению смешанного произведения и представлением векторного и скалярного произведений в координатной форме имеем:

    Векторы

    Полученная сумма произведений является расписанием определителя 3-го порядка, составленный из координат векторов, по элементам его третьей строки, то есть:

    Векторы

    Векторы Векторы компланарны тогда и только тогда, когда определитель 3-го порядка, элементами строк которого являются координаты этих векторов равен нулю (свойство 1):

    Векторы компланарны Векторы

    С помощью смешанного произведения векторов легко определить, относятся ли четыре точки Векторы одной плоскости. Для этого следует проверить выполнение условия компланарности трех векторов с общим началом в одной из точек.

    Простейшие задачи аналитической геометрии

    Задача об определении длины отрезка. Найти длину отрезка Векторы, если известны координаты его концов: Векторы. Эту задачу можно рассматривать как задачу о нахождении расстояния между двумя точками.

    1. Введем в рассмотрение вектор Векторы с началом Векторы и концом Векторы и радиусы-векторы ВекторыВекторы (рис. 6.13).
    2. Определим координаты вектора Векторы как разности векторов Векторыи Векторы: Векторы
    3. Находим модуль вектора Векторы, который и равна длине отрезка Векторы:

    Векторы

    Задача об определении площади треугольника

    Найдем площадь треугольника, заданного координатами вершин: ВекторыВекторы

    По аксиомой стереометрии известно, что три точки в пространстве определяют плоскость и притом только одну. Для упрощения изложения, не нарушает общего подхода к решению задачи, договоримся рассматривать треугольник Векторы, принадлежащей плоскости Векторы: Векторы и Векторы.

    1. Введем в рассмотрение векторы:

    Векторы

    и найдем их векторное произведение Векторы

    По соотношению (6.20) имеем: 

    Векторы

    2. Вычислим модуль вектора Векторы, численно равна площади параллелограмма Векторы, построенного на векторах Векторы как на сторонах (рис. 6.14):

    Векторы

    Тогда для площади треугольника Векторы имеем: 

    Векторы

    Знак Векторыили Векторы берется в зависимости от того, каким будет определитель – положительным или отрицательным.

    Если треугольник принадлежит не плоскости Векторы, а любой другой плоскости в пространстве, то его площадь тоже можно найти по формуле:

    Векторы

    Найдем площадь треугольника с вершинами Векторы Векторы Векторы

    Введем в рассмотрение векторы: Векторы и Векторы Векторы и определим их векторное произведение:

    Векторы

    Тогда 

    Векторы (кв. ед.)

    Задача о деление отрезка в заданном отношении

    Пусть в пространстве заданы две точки Векторы. Проведем через них произвольную прямую Векторы и установим на этой прямой положительное направление, согласно которому определим направление на отрезке Векторы (рис. 6.15). На прямой Векторы возьмем точку Векторы, которая может принадлежать отрезку Векторы, или его продолжению. При этом, если точка Векторы принадлежит отрезку Векторы (рис. 6.15 а), говорится, что она осуществляет внутреннее деление отрезка на части, если не принадлежит (рис. 6.15 б) – то внешний.

    Векторы

    Рис. 6.15

    Число Векторы, которое определяется формулой

    Векторы

    называется отношением, в котором точка Векторы разделяет направленный отрезок Векторы. Если Векторы, то Векторы осуществляет внутреннее (внешнее) деление отрезка на части.

    Задача о деление отрезка в заданном отношении формулируется так: найти координаты точки Векторы, что разделяет отрезок Векторы в отношении Векторы, если отрезок Векторы задан координатами начала Векторы и конца – Векторы

    Пусть точкам Векторы соответствуют радиусы-векторы Векторы (рис. 6.16). Из определения (6.29) следует, что векторы Векторы и Векторы коллинеарны, то есть Векторы. Следовательно, Векторы

    С этого векторного равенства найдем вектор Векторы

    Векторы

    или в координатах:

    Векторы

    Отсюда, если отрезок разделить на две равные части точкой Векторы то координаты точки Векторы могут быть найдены следующим образом:

    Векторы

    Можно доказать, что координаты точки пересечения медиан треугольника, заданного координатами его вершин Векторы вычисляются по формулам: 

    Векторы

    Векторы

    Векторы

    Лекции:

    • Объем конуса
    • Разложение на множители
    • Деление многочлена на многочлен
    • Правила дифференцирования
    • Теорема Пифагора
    • Асимптотическое поведение функций. Сравнение бесконечно малых функций
    • Прямая линия на плоскости
    • Выпуклость и вогнутость графика функции
    • Матанализ для чайников
    • Производные некоторых элементарных функций

    Для
    начала вспомним, какие действия над векторами вам известны.

    Итак,
    это сложение двух векторов по правилу треугольника или параллелограмма и
    нескольких векторов по правилу многоугольника. Вектор разности векторов  мы
    получали как вектор суммы векторов .

    Также
    вам знакомо правило умножение вектора на число.

    Сегодня
    вы познакомитесь с ещё одним действием над векторами — скалярным умножением
    векторов.

    Определение.
    Скалярным произведением двух векторов называется произведение их длин
    на косинус угла между ними.

    Скалярное
    произведение векторов  обозначают
    так .

    Или
    возможна запись без знака умножения.

    Оно
    равно произведению длин данных векторов на косинус угла между ними.

    Стоит
    вспомнить, что угол между векторами получают, откладывая данные векторы от
    одной точки. При этом выбирают угол меньший 180°

    Обратите
    внимание, ранее, при выполнении сложения, вычитания векторов и умножения
    вектора на число, результатом каждого из этих действий мы получали некоторый
    вектор.

    Результатом
    же скалярного произведения векторов является число.

    Сейчас
    подробнее рассмотрим случай, когда скалярное произведение векторов равно 0.

    Понятно,
    что для этого хотя бы один из множителей должен быть равен нулю.

    Такими
    будут случаи, когда хотя бы один из векторов в произведении является нулевым.

    Если
    же векторы  ненулевые,
    то косинус угла между ними должен быть равен 0.

    Среди
    возможных значений градусной меры угла между двумя векторами только лишь косинус
    угла в 90° равен 0.

    Отсюда
    получаем, что векторы  перпендикулярны.

    Подытожим.
    Скалярное произведение равно нулю, если хотя бы один из векторов сомножителей является
    нулевым.

    Ну,
    а скалярное произведение ненулевых векторов равно нулю тогда и только
    тогда, когда эти векторы перпендикулярны.

    Из
    формулы скалярного произведения также можно заметить, что, если векторы  не
    нулевые, то их длины всегда больше нуля, поэтому их произведение тоже
    положительно. А вот значение косинуса угла между ними может принимать как
    положительные, так и отрицательные значения.

    Можно
    сказать, что скалярное произведение двух ненулевых векторов больше нуля, если
    угол между векторами острый. Равно нулю, если угол между ним прямой. И меньше
    нуля, если угол между данными векторами тупой. Ещё раз обратим внимание на то,
    что эти заключения верны для ненулевых векторов .

    Задача.
    Найти скалярное произведение векторов  и
    ,
    пользуясь данными рисунков.

    Решение.

    а)

    б)

    в)

    г)

    Мы
    рассмотрели примеры применения формулы скалярного произведения двух векторов и
    убедились, что скалярное произведение ненулевых векторов больше нуля, если угол
    между ними является острым, равно нулю — если векторы перпендикулярны, и меньше
    нуля — если угол между векторами тупой.

    А
    сейчас рассмотрим сонаправленные векторы  и
    .
    Запишем формулу их скалярного произведения.

    Вы
    должны помнить с прошлых уроков, что угол между сонаправленными векторами равен
    нулю. А косинус угла в 0° равен 1. Тогда получаем, что скалярное произведение
    сонаправленных векторов равно произведению их длин.

    Говоря
    о противоположно направленных векторах, можно вспомнить, что угол между ними
    равен 180°. Значит, косинус равен -1.

    Тогда
    скалярное произведение противоположно направленных векторов равно .

    Что
    касается, скалярного произведения вектора на самого себя, то его называют скалярным
    квадратом вектора.
    Этот случай можно рассматривать в контексте
    сонаправленных векторов. Действительно, ведь векторы равны, а значит, и
    сонаправлены. Такое произведение равно произведению длин данного вектора.

    Тогда
    получаем, что скалярный квадрат вектора равен квадрату его длины.

    Задача.
    Найдём скалярные квадраты векторов ,
    ,
     и
    .

    Решение.

     

     

     

     

    Скалярное
    произведение векторов применяется не только в математике. Например, из курса
    механики известно, что работа постоянной силы F
    при перемещении из точки М в точку Н равна .

    Тем
    самым получаем, что работа силы F
    равна скалярному произведению вектора силы  и
    вектора перемещения .

    Вернёмся
    к скалярному произведению в математике и решим несколько задач.

    Задача.
    К одной и той же точке приложены  и
    ,
    действующие под углом в  друг
    к другу. ,
    .
    Найти величину равнодействующей силы .

    Решение.

    1
    способ

     

     

     

     

     

    ,

    2
    способ

     

     

     

     

     

    ,

    Ответ:
    .

    Задача.
    В ,где
    ,
    проведена высота .
    Вычислить ,
    ,
    ,
    .

    Решение

     

     

    ,
    так как

     

     

    Подведём
    итоги нашего урока.

    Сегодня
    вы познакомились с новым действием над векторами — скалярным умножением
    векторов.

    Скалярным
    произведением двух векторов называют произведение длин данных векторов на
    косинус угла между ними.

    Проанализировав
    эту формулу, мы заметили, что скалярное произведение равно нулю, если хотя бы
    один из векторов сомножителей является нулевым. Ну, а скалярное произведение
    ненулевых векторов рано нулю, тогда и только тогда, когда данные векторы
    перпендикулярны.

    Также,
    пользуясь знаниями об углах между сонаправленными и противоположно
    направленными векторами, мы выяснили, что скалярное произведение сонаправленных
    векторов равно произведению их длин, а скалярное произведение противоположно
    направленных векторов противоположно произведению их длин.

    Введя
    понятия скалярного квадрата вектора, мы получили, что он равен квадрату длины
    данного вектора.

    Знания
    о скалярном произведении векторов можно применять не только на уроках
    математики. Так же они широко используются в физике.

    Два вектора

    a→

    и

    b→

     всегда образуют угол.

    Угол между векторами может принимать значения от

    до

    180°

    включительно.

    Если векторы не параллельны, то их можно расположить на пересекающихся прямых.

    Векторы могут образовать:

    1. острый угол;

    Lenkis_vekt4.png

    2. тупой угол;

    Lenkis_vekt5.png

    3. прямой угол (векторы перпендикулярны).

    Lenkis_vekt2.png

    Если векторы расположены на параллельных прямых, то они могут образовать:

    4. угол величиной

     (векторы сонаправлены);

    Lenkis_vekt1.png

    5. угол величиной

    180°

     (векторы противоположно направлены).

    Lenkis_vekt3.png

    Если один из векторов или оба вектора нулевые, то угол между ними будет равен

    .

    Угол между векторами записывают так:

    Скалярное произведение векторов

    Скалярным произведением двух векторов называется число, равное произведению длин этих векторов на косинус угла между ними:

    a→⋅b→=a→⋅b→⋅cosa→b→ˆ

    .

    Результат скалярного произведения векторов является числом (в отличие от результата рассмотренных ранее действий с векторами — сложения, вычитания и умножения на число. В таких случаях результатом был вектор). При умножении вектора на вектор получается число, так как длины векторов — это числа, косинус угла — число — соответственно, их произведение также будет являться числом.

    1. Если угол между векторами острый, то скалярное произведение будет положительным числом (так как косинус острого угла — положительное число). 

    Если векторы сонаправлены, то угол между ними будет равен

    , а косинус равен (1), скалярное произведение также будет положительным.

    2. Если угол между векторами тупой, то скалярное произведение будет отрицательным (так как косинус тупого угла — отрицательное число). 

    Если векторы направлены противоположно, то угол между ними будет равен

    180°

    . Скалярное произведение также отрицательно, так как косинус этого угла равен (-1).

    Справедливы и обратные утверждения:

    1. Если скалярное произведение векторов — положительное число, то угол между данными векторами острый.

    2. Если скалярное произведение векторов — отрицательное число, то угол между данными векторами тупой.

    Особенный третий случай!

    Обрати внимание!

    3. Если угол между векторами прямой, то скалярное произведение векторов равно нулю, так как косинус прямого угла равен (0).

    Обратное суждение: если скалярное произведение векторов равно нулю, то эти векторы перпендикулярны.

    Вектор, умноженный на самого себя, будет числом, которое называется скалярным квадратом вектора. Скалярный квадрат вектора  равен квадрату длины данного вектора и обозначается как 

    a→2

    .

    Свойства скалярного произведения

    Для любых векторов и любого числа справедливы следующие свойства:

    1.

    a→2≥0

    , к тому же

    a→2>0

    , если

    a→≠0→

    .

    2. Переместительный, или коммутативный, закон скалярного произведения:

    a→⋅b→=b→⋅a→

    .

    3. Распределительный, или дистрибутивный, закон скалярного произведения:

    a→+b→⋅c→=a→⋅c→+b→⋅c→

    .

    4. Сочетательный, или ассоциативный, закон скалярного произведения:

    k⋅a→⋅b→=k⋅a→⋅b→

    .

    Использование скалярного произведения

    Удобно использовать скалярное произведение векторов для определения углов между прямыми и между прямой и плоскостью.

    Угол между прямыми

    Ознакомимся с ещё одним определением.

    Вектор называют направляющим вектором прямой, если он находится на прямой или параллелен этой прямой.

    Taisne_vektors.png

    Чтобы определить косинус угла между прямыми, надо определить косинус угла между направляющими векторами этих прямых, то есть найти векторы, параллельные прямым, и определить косинус угла между векторами.

    Для этого необходимо рассмотреть определение скалярного произведения, если векторы даны в координатной системе.

    Если

    a→x1;y1;z1

    ,

    b→x2;y2;z2

    , то

    a→⋅b→=x1⋅x2+y1⋅y2+z1⋅z2

    .

    Прежде была рассмотрена формула определения длины вектора в координатной форме.

    Теперь, объединив эти формулы, получим формулу для определения косинуса угла между векторами в координатной форме. Так как из формулы скалярного произведения следует, что

    cosα=a→⋅b→a→⋅b→

    , то

    cosα=x1⋅x2+y1⋅y2+z1⋅z2x12+y12+z12 ⋅x22+y22+z22

    .

    Угол между прямой и плоскостью

    Введём понятие о нормальном векторе плоскости.

    Нормальный вектор плоскости — это любой ненулевой вектор, лежащий на прямой, перпендикулярной к данной плоскости.

    Plakne_vektors.png

    Используя следующий рисунок, легко доказать, что косинус угла

    β

    между нормальным вектором

    n→

     данной плоскости и неким вектором

    b→

     равен синусу угла

    α

    между прямой и плоскостью, так как

    α

    и

    β

     вместе образуют угол в

    90°

    .

    Plakne_vektors_lenkis.png

    При нахождении косинуса угла между

    n→

    и

    b→

    можно использовать это число как синус угла между прямой, на которой лежит вектор

    b→

    , и плоскостью.

    Добавить комментарий